Sunteți pe pagina 1din 143

The Physics GRE Solution Guide

GR0177 Test

FT
RA
http://groups.yahoo.com/group/physicsgre_v2

November 3, 2009
D

Author:
David S. Latchman
2

FT
RA
D

David S. Latchman ©2009


Preface

This solution guide initially started out on the Yahoo Groups web site and was pretty

FT
successful at the time. Unfortunately, the group was lost and with it, much of the the
hard work that was put into it. This is my attempt to recreate the solution guide and
make it more widely avaialble to everyone. If you see any errors, think certain things
could be expressed more clearly, or would like to make suggestions, please feel free to
do so.
David Latchman
RA
Document Changes
05-11-2009 1. Added diagrams to GR0177 test questions 1-25
2. Revised solutions to GR0177 questions 1-25

04-15-2009 First Version


D
ii

FT
RA
D

David S. Latchman ©2009


Contents

FT
Preface i

1 Classical Mechanics 1
1.1 Kinematics . . . . . . . . . . . . . . . . . . . . . . . . . . . . . . . . . . . . 1
1.2 Newton’s Laws . . . . . . . . . . . . . . . . . . . . . . . . . . . . . . . . . 2
1.3 Work & Energy . . . . . . . . . . . . . . . . . . . . . . . . . . . . . . . . . 3
RA
1.4 Oscillatory Motion . . . . . . . . . . . . . . . . . . . . . . . . . . . . . . . 4
1.5 Rotational Motion about a Fixed Axis . . . . . . . . . . . . . . . . . . . . 8
1.6 Dynamics of Systems of Particles . . . . . . . . . . . . . . . . . . . . . . . 10
1.7 Central Forces and Celestial Mechanics . . . . . . . . . . . . . . . . . . . 10
1.8 Three Dimensional Particle Dynamics . . . . . . . . . . . . . . . . . . . . 12
1.9 Fluid Dynamics . . . . . . . . . . . . . . . . . . . . . . . . . . . . . . . . . 12
1.10 Non-inertial Reference Frames . . . . . . . . . . . . . . . . . . . . . . . . 13
D

1.11 Hamiltonian and Lagrangian Formalism . . . . . . . . . . . . . . . . . . . 13

2 Electromagnetism 15
2.1 Electrostatics . . . . . . . . . . . . . . . . . . . . . . . . . . . . . . . . . . . 15
2.2 Currents and DC Circuits . . . . . . . . . . . . . . . . . . . . . . . . . . . 20
2.3 Magnetic Fields in Free Space . . . . . . . . . . . . . . . . . . . . . . . . . 20
2.4 Lorentz Force . . . . . . . . . . . . . . . . . . . . . . . . . . . . . . . . . . 20
2.5 Induction . . . . . . . . . . . . . . . . . . . . . . . . . . . . . . . . . . . . . 20
2.6 Maxwell’s Equations and their Applications . . . . . . . . . . . . . . . . . 20
2.7 Electromagnetic Waves . . . . . . . . . . . . . . . . . . . . . . . . . . . . . 20
iv Contents
2.8 AC Circuits . . . . . . . . . . . . . . . . . . . . . . . . . . . . . . . . . . . 20
2.9 Magnetic and Electric Fields in Matter . . . . . . . . . . . . . . . . . . . . 20
2.10 Capacitance . . . . . . . . . . . . . . . . . . . . . . . . . . . . . . . . . . . 21
2.11 Energy in a Capacitor . . . . . . . . . . . . . . . . . . . . . . . . . . . . . . 21
2.12 Energy in an Electric Field . . . . . . . . . . . . . . . . . . . . . . . . . . . 21
2.13 Current . . . . . . . . . . . . . . . . . . . . . . . . . . . . . . . . . . . . . . 21
2.14 Current Destiny . . . . . . . . . . . . . . . . . . . . . . . . . . . . . . . . . 21
2.15 Current Density of Moving Charges . . . . . . . . . . . . . . . . . . . . . 21
2.16 Resistance and Ohm’s Law . . . . . . . . . . . . . . . . . . . . . . . . . . 21
2.17 Resistivity and Conductivity . . . . . . . . . . . . . . . . . . . . . . . . . . 22

FT
2.18 Power . . . . . . . . . . . . . . . . . . . . . . . . . . . . . . . . . . . . . . . 22
2.19 Kirchoff’s Loop Rules . . . . . . . . . . . . . . . . . . . . . . . . . . . . . . 22
2.20 Kirchoff’s Junction Rule . . . . . . . . . . . . . . . . . . . . . . . . . . . . 22
2.21 RC Circuits . . . . . . . . . . . . . . . . . . . . . . . . . . . . . . . . . . . . 22
2.22 Maxwell’s Equations . . . . . . . . . . . . . . . . . . . . . . . . . . . . . . 22
RA
2.23 Speed of Propagation of a Light Wave . . . . . . . . . . . . . . . . . . . . 23
2.24 Relationship between E and B Fields . . . . . . . . . . . . . . . . . . . . . 23
2.25 Energy Density of an EM wave . . . . . . . . . . . . . . . . . . . . . . . . 24
2.26 Poynting’s Vector . . . . . . . . . . . . . . . . . . . . . . . . . . . . . . . . 24

3 Optics & Wave Phonomena 25


3.1 Wave Properties . . . . . . . . . . . . . . . . . . . . . . . . . . . . . . . . . 25
3.2 Superposition . . . . . . . . . . . . . . . . . . . . . . . . . . . . . . . . . . 25
D

3.3 Interference . . . . . . . . . . . . . . . . . . . . . . . . . . . . . . . . . . . 25
3.4 Diffraction . . . . . . . . . . . . . . . . . . . . . . . . . . . . . . . . . . . . 25
3.5 Geometrical Optics . . . . . . . . . . . . . . . . . . . . . . . . . . . . . . . 25
3.6 Polarization . . . . . . . . . . . . . . . . . . . . . . . . . . . . . . . . . . . 25
3.7 Doppler Effect . . . . . . . . . . . . . . . . . . . . . . . . . . . . . . . . . . 26
3.8 Snell’s Law . . . . . . . . . . . . . . . . . . . . . . . . . . . . . . . . . . . . 26

4 Thermodynamics & Statistical Mechanics 27


4.1 Laws of Thermodynamics . . . . . . . . . . . . . . . . . . . . . . . . . . . 27
4.2 Thermodynamic Processes . . . . . . . . . . . . . . . . . . . . . . . . . . . 27

David S. Latchman ©2009


Contents v
4.3 Equations of State . . . . . . . . . . . . . . . . . . . . . . . . . . . . . . . . 27
4.4 Ideal Gases . . . . . . . . . . . . . . . . . . . . . . . . . . . . . . . . . . . . 27
4.5 Kinetic Theory . . . . . . . . . . . . . . . . . . . . . . . . . . . . . . . . . . 27
4.6 Ensembles . . . . . . . . . . . . . . . . . . . . . . . . . . . . . . . . . . . . 27
4.7 Statistical Concepts and Calculation of Thermodynamic Properties . . . 28
4.8 Thermal Expansion & Heat Transfer . . . . . . . . . . . . . . . . . . . . . 28
4.9 Heat Capacity . . . . . . . . . . . . . . . . . . . . . . . . . . . . . . . . . . 28
4.10 Specific Heat Capacity . . . . . . . . . . . . . . . . . . . . . . . . . . . . . 28
4.11 Heat and Work . . . . . . . . . . . . . . . . . . . . . . . . . . . . . . . . . 28
4.12 First Law of Thermodynamics . . . . . . . . . . . . . . . . . . . . . . . . . 28

FT
4.13 Work done by Ideal Gas at Constant Temperature . . . . . . . . . . . . . 29
4.14 Heat Conduction Equation . . . . . . . . . . . . . . . . . . . . . . . . . . . 29
4.15 Ideal Gas Law . . . . . . . . . . . . . . . . . . . . . . . . . . . . . . . . . . 30
4.16 Stefan-Boltzmann’s FormulaStefan-Boltzmann’s Equation . . . . . . . . 30
4.17 RMS Speed of an Ideal Gas . . . . . . . . . . . . . . . . . . . . . . . . . . 30
RA
4.18 Translational Kinetic Energy . . . . . . . . . . . . . . . . . . . . . . . . . . 30
4.19 Internal Energy of a Monatomic gas . . . . . . . . . . . . . . . . . . . . . 30
4.20 Molar Specific Heat at Constant Volume . . . . . . . . . . . . . . . . . . . 31
4.21 Molar Specific Heat at Constant Pressure . . . . . . . . . . . . . . . . . . 31
4.22 Equipartition of Energy . . . . . . . . . . . . . . . . . . . . . . . . . . . . 31
4.23 Adiabatic Expansion of an Ideal Gas . . . . . . . . . . . . . . . . . . . . . 33
4.24 Second Law of Thermodynamics . . . . . . . . . . . . . . . . . . . . . . . 33
D

5 Quantum Mechanics 35
5.1 Fundamental Concepts . . . . . . . . . . . . . . . . . . . . . . . . . . . . . 35
5.2 Schrödinger Equation . . . . . . . . . . . . . . . . . . . . . . . . . . . . . . 35
5.3 Spin . . . . . . . . . . . . . . . . . . . . . . . . . . . . . . . . . . . . . . . . 40
5.4 Angular Momentum . . . . . . . . . . . . . . . . . . . . . . . . . . . . . . 41
5.5 Wave Funtion Symmetry . . . . . . . . . . . . . . . . . . . . . . . . . . . . 41
5.6 Elementary Perturbation Theory . . . . . . . . . . . . . . . . . . . . . . . 41

6 Atomic Physics 43
6.1 Properties of Electrons . . . . . . . . . . . . . . . . . . . . . . . . . . . . . 43

©2009 David S. Latchman


vi Contents
6.2 Bohr Model . . . . . . . . . . . . . . . . . . . . . . . . . . . . . . . . . . . 43
6.3 Energy Quantization . . . . . . . . . . . . . . . . . . . . . . . . . . . . . . 44
6.4 Atomic Structure . . . . . . . . . . . . . . . . . . . . . . . . . . . . . . . . 44
6.5 Atomic Spectra . . . . . . . . . . . . . . . . . . . . . . . . . . . . . . . . . 45
6.6 Selection Rules . . . . . . . . . . . . . . . . . . . . . . . . . . . . . . . . . . 45
6.7 Black Body Radiation . . . . . . . . . . . . . . . . . . . . . . . . . . . . . . 45
6.8 X-Rays . . . . . . . . . . . . . . . . . . . . . . . . . . . . . . . . . . . . . . 46
6.9 Atoms in Electric and Magnetic Fields . . . . . . . . . . . . . . . . . . . . 47

7 Special Relativity 51

FT
7.1 Introductory Concepts . . . . . . . . . . . . . . . . . . . . . . . . . . . . . 51
7.2 Time Dilation . . . . . . . . . . . . . . . . . . . . . . . . . . . . . . . . . . 51
7.3 Length Contraction . . . . . . . . . . . . . . . . . . . . . . . . . . . . . . . 51
7.4 Simultaneity . . . . . . . . . . . . . . . . . . . . . . . . . . . . . . . . . . . 52
7.5 Energy and Momentum . . . . . . . . . . . . . . . . . . . . . . . . . . . . 52
7.6 Four-Vectors and Lorentz Transformation . . . . . . . . . . . . . . . . . . 53
RA
7.7 Velocity Addition . . . . . . . . . . . . . . . . . . . . . . . . . . . . . . . . 54
7.8 Relativistic Doppler Formula . . . . . . . . . . . . . . . . . . . . . . . . . 54
7.9 Lorentz Transformations . . . . . . . . . . . . . . . . . . . . . . . . . . . . 55
7.10 Space-Time Interval . . . . . . . . . . . . . . . . . . . . . . . . . . . . . . . 55

8 Laboratory Methods 57
8.1 Data and Error Analysis . . . . . . . . . . . . . . . . . . . . . . . . . . . . 57
D

8.2 Instrumentation . . . . . . . . . . . . . . . . . . . . . . . . . . . . . . . . . 59
8.3 Radiation Detection . . . . . . . . . . . . . . . . . . . . . . . . . . . . . . . 59
8.4 Counting Statistics . . . . . . . . . . . . . . . . . . . . . . . . . . . . . . . 59
8.5 Interaction of Charged Particles with Matter . . . . . . . . . . . . . . . . 60
8.6 Lasers and Optical Interferometers . . . . . . . . . . . . . . . . . . . . . . 60
8.7 Dimensional Analysis . . . . . . . . . . . . . . . . . . . . . . . . . . . . . 60
8.8 Fundamental Applications of Probability and Statistics . . . . . . . . . . 60

9 GR0177 Exam Solutions 61


9.1 Acceleration of a Pendulum Bob . . . . . . . . . . . . . . . . . . . . . . . 61

David S. Latchman ©2009


Contents vii
9.2 Coin on a Turntable . . . . . . . . . . . . . . . . . . . . . . . . . . . . . . . 62
9.3 Kepler’s Law and Satellite Orbits . . . . . . . . . . . . . . . . . . . . . . . 63
9.4 Non-Elastic Collisions . . . . . . . . . . . . . . . . . . . . . . . . . . . . . 64
9.5 The Equipartition Theorem and the Harmonic Oscillator . . . . . . . . . 65
9.6 Work Done in Isothermal and Adiabatic Expansions . . . . . . . . . . . . 65
9.7 Electromagnetic Field Lines . . . . . . . . . . . . . . . . . . . . . . . . . . 67
9.8 Image Charges . . . . . . . . . . . . . . . . . . . . . . . . . . . . . . . . . . 67
9.9 Electric Field Symmetry . . . . . . . . . . . . . . . . . . . . . . . . . . . . 68
9.10 Networked Capacitors . . . . . . . . . . . . . . . . . . . . . . . . . . . . . 68
9.11 Thin Lens Equation . . . . . . . . . . . . . . . . . . . . . . . . . . . . . . . 69

FT
9.12 Mirror Equation . . . . . . . . . . . . . . . . . . . . . . . . . . . . . . . . . 70
9.13 Resolving Power of a Telescope . . . . . . . . . . . . . . . . . . . . . . . . 70
9.14 Radiation detected by a NaI(Tl) crystal . . . . . . . . . . . . . . . . . . . . 71
9.15 Accuracy and Precision . . . . . . . . . . . . . . . . . . . . . . . . . . . . . 72
9.16 Counting Statistics . . . . . . . . . . . . . . . . . . . . . . . . . . . . . . . 72
RA
9.17 Electron configuration . . . . . . . . . . . . . . . . . . . . . . . . . . . . . 73
9.18 Ionization Potential (He atom) . . . . . . . . . . . . . . . . . . . . . . . . . 73
9.19 Nuclear Fusion . . . . . . . . . . . . . . . . . . . . . . . . . . . . . . . . . 74
9.20 Bremsstrahlung X-Rays . . . . . . . . . . . . . . . . . . . . . . . . . . . . . 74
9.21 Atomic Spectra . . . . . . . . . . . . . . . . . . . . . . . . . . . . . . . . . 74
9.22 Planetary Orbits . . . . . . . . . . . . . . . . . . . . . . . . . . . . . . . . . 75
9.23 Acceleration of particle in circular motion . . . . . . . . . . . . . . . . . . 76
D

9.24 Two-Dimensional Trajectories . . . . . . . . . . . . . . . . . . . . . . . . . 77


9.25 Moment of inertia of pennies in a circle . . . . . . . . . . . . . . . . . . . 77
9.26 Falling Rod . . . . . . . . . . . . . . . . . . . . . . . . . . . . . . . . . . . . 78
9.27 Hermitian Operator . . . . . . . . . . . . . . . . . . . . . . . . . . . . . . . 79
9.28 Orthogonality . . . . . . . . . . . . . . . . . . . . . . . . . . . . . . . . . . 79
9.29 Expectation Values . . . . . . . . . . . . . . . . . . . . . . . . . . . . . . . 80
9.30 Radial Wave Functions . . . . . . . . . . . . . . . . . . . . . . . . . . . . . 80
9.31 Decay of Positronium Atom . . . . . . . . . . . . . . . . . . . . . . . . . . 81
9.32 Relativistic Energy and Momentum . . . . . . . . . . . . . . . . . . . . . 81
9.33 Speed of a Charged pion . . . . . . . . . . . . . . . . . . . . . . . . . . . . 82

©2009 David S. Latchman


viii Contents
9.34 Simultaneity . . . . . . . . . . . . . . . . . . . . . . . . . . . . . . . . . . . 82
9.35 Black-Body Radiation . . . . . . . . . . . . . . . . . . . . . . . . . . . . . . 83
9.36 Quasi-static Adiabatic Expansion of an Ideal Gas . . . . . . . . . . . . . . 83
9.37 Thermodynamic Cycles . . . . . . . . . . . . . . . . . . . . . . . . . . . . 84
9.38 RLC Resonant Circuits . . . . . . . . . . . . . . . . . . . . . . . . . . . . . 85
9.39 High Pass Filters . . . . . . . . . . . . . . . . . . . . . . . . . . . . . . . . 86
9.40 RL Circuits . . . . . . . . . . . . . . . . . . . . . . . . . . . . . . . . . . . . 87
9.41 Maxwell’s Equations . . . . . . . . . . . . . . . . . . . . . . . . . . . . . . 88
9.42 Faraday’s Law of Induction . . . . . . . . . . . . . . . . . . . . . . . . . . 89
9.43 Quantum Mechanics: Commutators . . . . . . . . . . . . . . . . . . . . . 89

FT
9.44 Energies . . . . . . . . . . . . . . . . . . . . . . . . . . . . . . . . . . . . . 90
9.45 1-D Harmonic Oscillator . . . . . . . . . . . . . . . . . . . . . . . . . . . . 90
9.46 de Broglie Wavelength . . . . . . . . . . . . . . . . . . . . . . . . . . . . . 91
9.47 Entropy . . . . . . . . . . . . . . . . . . . . . . . . . . . . . . . . . . . . . . 92
9.48 RMS Speed . . . . . . . . . . . . . . . . . . . . . . . . . . . . . . . . . . . . 92
RA
9.49 Partition Function . . . . . . . . . . . . . . . . . . . . . . . . . . . . . . . . 93
9.50 Resonance of an Open Cylinder . . . . . . . . . . . . . . . . . . . . . . . . 93
9.51 Polarizers . . . . . . . . . . . . . . . . . . . . . . . . . . . . . . . . . . . . . 94
9.52 Crystallography . . . . . . . . . . . . . . . . . . . . . . . . . . . . . . . . . 94
9.53 Resistance of a Semiconductor . . . . . . . . . . . . . . . . . . . . . . . . . 94
9.54 Impulse . . . . . . . . . . . . . . . . . . . . . . . . . . . . . . . . . . . . . . 95
9.55 Fission Collision . . . . . . . . . . . . . . . . . . . . . . . . . . . . . . . . . 95
D

9.56 Archimedes’ Principal and Buoyancy . . . . . . . . . . . . . . . . . . . . 96


9.57 Fluid Dynamics . . . . . . . . . . . . . . . . . . . . . . . . . . . . . . . . . 97
9.58 Charged Particle in an EM-field . . . . . . . . . . . . . . . . . . . . . . . . 97
9.59 LC Circuits and Mechanical Oscillators . . . . . . . . . . . . . . . . . . . 98
9.60 Gauss’ Law . . . . . . . . . . . . . . . . . . . . . . . . . . . . . . . . . . . . 99
9.61 Electromagnetic Boundary Conditions . . . . . . . . . . . . . . . . . . . . 99
9.62 Cyclotron Frequency . . . . . . . . . . . . . . . . . . . . . . . . . . . . . . 99
9.63 Wein’s Law . . . . . . . . . . . . . . . . . . . . . . . . . . . . . . . . . . . . 100
9.64 Electromagnetic Spectra . . . . . . . . . . . . . . . . . . . . . . . . . . . . 100
9.65 Molar Heat Capacity . . . . . . . . . . . . . . . . . . . . . . . . . . . . . . 101

David S. Latchman ©2009


Contents ix
9.66 Radioactive Decay . . . . . . . . . . . . . . . . . . . . . . . . . . . . . . . 101
9.67 Nuclear Binding Energy . . . . . . . . . . . . . . . . . . . . . . . . . . . . 102
9.68 Radioactive Decay . . . . . . . . . . . . . . . . . . . . . . . . . . . . . . . 103
9.69 Thin Film Interference . . . . . . . . . . . . . . . . . . . . . . . . . . . . . 103
9.70 Double Slit Experiment . . . . . . . . . . . . . . . . . . . . . . . . . . . . . 103
9.71 Atomic Spectra and Doppler Red Shift . . . . . . . . . . . . . . . . . . . . 104
9.72 Springs, Forces and Falling Masses . . . . . . . . . . . . . . . . . . . . . . 104
9.73 Blocks and Friction . . . . . . . . . . . . . . . . . . . . . . . . . . . . . . . 104
9.74 Lagrangians . . . . . . . . . . . . . . . . . . . . . . . . . . . . . . . . . . . 105
9.75 Matrix Transformations & Rotations . . . . . . . . . . . . . . . . . . . . . 106

FT
9.76 Fermi Gases . . . . . . . . . . . . . . . . . . . . . . . . . . . . . . . . . . . 106
9.77 Maxwell-Boltzmann Distributions . . . . . . . . . . . . . . . . . . . . . . 106
9.78 Conservation of Lepton Number and Muon Decay . . . . . . . . . . . . . 107
9.79 Rest Mass of a Particle . . . . . . . . . . . . . . . . . . . . . . . . . . . . . 108
9.80 Relativistic Addition of Velocities . . . . . . . . . . . . . . . . . . . . . . . 108
RA
9.81 Angular Momentum . . . . . . . . . . . . . . . . . . . . . . . . . . . . . . 108
9.82 Addition of Angular Momentum . . . . . . . . . . . . . . . . . . . . . . . 109
9.83 Spin Basises . . . . . . . . . . . . . . . . . . . . . . . . . . . . . . . . . . . 109
9.84 Selection Rules . . . . . . . . . . . . . . . . . . . . . . . . . . . . . . . . . . 109
9.85 Resistivity . . . . . . . . . . . . . . . . . . . . . . . . . . . . . . . . . . . . 110
9.86 Faraday’s Law . . . . . . . . . . . . . . . . . . . . . . . . . . . . . . . . . . 111
9.87 Electric Potential . . . . . . . . . . . . . . . . . . . . . . . . . . . . . . . . 112
D

9.88 Biot-Savart Law . . . . . . . . . . . . . . . . . . . . . . . . . . . . . . . . . 112


9.89 Conservation of Angular Momentum . . . . . . . . . . . . . . . . . . . . 113
9.90 Springs in Series and Parallel . . . . . . . . . . . . . . . . . . . . . . . . . 114
9.91 Cylinder rolling down an incline . . . . . . . . . . . . . . . . . . . . . . . 115
9.92 Hamiltonian of Mass-Spring System . . . . . . . . . . . . . . . . . . . . . 116
9.93 Radius of the Hydrogen Atom . . . . . . . . . . . . . . . . . . . . . . . . . 116
9.94 Perturbation Theory . . . . . . . . . . . . . . . . . . . . . . . . . . . . . . 117
9.95 Electric Field in a Dielectric . . . . . . . . . . . . . . . . . . . . . . . . . . 117
9.96 EM Radiation . . . . . . . . . . . . . . . . . . . . . . . . . . . . . . . . . . 117
9.97 Dispersion of a Light Beam . . . . . . . . . . . . . . . . . . . . . . . . . . 117

©2009 David S. Latchman


x Contents
9.98 Average Energy of a Thermal System . . . . . . . . . . . . . . . . . . . . . 118
9.99 Pair Production in vincinity of an electron . . . . . . . . . . . . . . . . . . 118
9.100Michelson Interferometer . . . . . . . . . . . . . . . . . . . . . . . . . . . 120

A Constants & Important Equations 121


A.1 Constants . . . . . . . . . . . . . . . . . . . . . . . . . . . . . . . . . . . . . 121
A.2 Vector Identities . . . . . . . . . . . . . . . . . . . . . . . . . . . . . . . . . 121
A.3 Commutators . . . . . . . . . . . . . . . . . . . . . . . . . . . . . . . . . . 122
A.4 Linear Algebra . . . . . . . . . . . . . . . . . . . . . . . . . . . . . . . . . . 123

FT
RA
D

David S. Latchman ©2009


List of Tables

FT
4.22.1Table of Molar Specific Heats . . . . . . . . . . . . . . . . . . . . . . . . . 32

A.1.1Something . . . . . . . . . . . . . . . . . . . . . . . . . . . . . . . . . . . . 121
RA
D
xii List of Tables

FT
RA
D

David S. Latchman ©2009


List of Figures

FT
9.1.1 Acceleration components on pendulum bob . . . . . . . . . . . . . . . . . 61
9.1.2 Acceleration vectors of bob at equilibrium and max. aplitude positions . 62
9.2.1 Free Body Diagram of Coin on Turn-Table . . . . . . . . . . . . . . . . . . 62
9.4.1 Inelastic collision between masses 2m and m . . . . . . . . . . . . . . . . 64
9.9.1 Five charges arranged symmetrically around circle of radius, r . . . . . . 68
9.10.1Capacitors in series and its equivalent circuit . . . . . . . . . . . . . . . . 68
RA
9.14.1Diagram of NaI(Tl) detector postions . . . . . . . . . . . . . . . . . . . . . 71
9.23.1Acceleration components of a particle moving in circular motion . . . . 76
9.25.1Seven pennies in a hexagonal, planar pattern . . . . . . . . . . . . . . . . 77
9.26.1Falling rod attached to a pivot point . . . . . . . . . . . . . . . . . . . . . 78
9.56.1Diagram of Helium filled balloon attached to a mass . . . . . . . . . . . . 96
D
xiv List of Figures

FT
RA
D

David S. Latchman ©2009


Chapter 1
Classical Mechanics

1.1

1.1.1
Kinematics

Linear Motion

Average Velocity FT
RA
∆x x2 − x1
v= = (1.1.1)
∆t t2 − t1

Instantaneous Velocity

∆x dx
v = lim = = v(t) (1.1.2)
∆t→0 ∆t dt

Kinematic Equations of Motion


D

The basic kinematic equations of motion under constant acceleration, a, are


v = v0 + at (1.1.3)
v2 = v20 + 2a (x − x0 ) (1.1.4)
1
x − x0 = v0 t + at2 (1.1.5)
2
1
x − x0 = (v + v0 ) t (1.1.6)
2

1.1.2 Circular Motion

In the case of Uniform Circular Motion, for a particle to move in a circular path, a
radial acceleration must be applied. This acceleration is known as the Centripetal
2 Classical Mechanics
Acceleration

Centripetal Acceleration

v2
a= (1.1.7)
r

Angular Velocity

v
ω= (1.1.8)
r
We can write eq. (1.1.7) in terms of ω

FT
a = ω2 r (1.1.9)

Rotational Equations of Motion

The equations of motion under a constant angular acceleration, α, are

ω = ω0 + αt (1.1.10)
RA
ω + ω0
θ= t (1.1.11)
2
1
θ = ω0 t + αt2 (1.1.12)
2
ω2 = ω20 + 2αθ (1.1.13)

1.2 Newton’s Laws


D

1.2.1 Newton’s Laws of Motion

First Law A body continues in its state of rest or of uniform motion unless acted upon
by an external unbalanced force.

Second Law The net force on a body is proportional to its rate of change of momentum.

dp
F= = ma (1.2.1)
dt

Third Law When a particle A exerts a force on another particle B, B simultaneously


exerts a force on A with the same magnitude in the opposite direction.

FAB = −FBA (1.2.2)

David S. Latchman ©2009


Work & Energy 3
1.2.2 Momentum
p = mv (1.2.3)

1.2.3 Impulse
w
∆p = J = Fdt = Favg dt (1.2.4)

1.3 Work & Energy

1.3.1 Kinetic Energy

FT
1
K ≡ mv2 (1.3.1)
2

1.3.2 The Work-Energy Theorem


The net Work done is given by
Wnet = K f − Ki (1.3.2)
RA
1.3.3 Work done under a constant Force
The work done by a force can be expressed as
W = F∆x (1.3.3)
In three dimensions, this becomes
W = F · ∆r = F∆r cos θ (1.3.4)
For a non-constant force, we have
D

wx f
W= F(x)dx (1.3.5)
xi

1.3.4 Potential Energy


The Potential Energy is
dU(x)
F(x) = − (1.3.6)
dx
for conservative forces, the potential energy is
wx
U(x) = U0 − F(x0 )dx0 (1.3.7)
x0

©2009 David S. Latchman


4 Classical Mechanics
1.3.5 Hooke’s Law

F = −kx (1.3.8)
where k is the spring constant.

1.3.6 Potential Energy of a Spring

1
U(x) = kx2 (1.3.9)
2

1.4 Oscillatory Motion

1.4.1 Equation for Simple Harmonic Motion

FT
x(t) = A sin (ωt + δ) (1.4.1)
where the Amplitude, A, measures the displacement from equilibrium, the phase, δ, is
the angle by which the motion is shifted from equilibrium at t = 0.
RA
1.4.2 Period of Simple Harmonic Motion


T= (1.4.2)
ω

1.4.3 Total Energy of an Oscillating System


D

Given that
x = A sin (ωt + δ) (1.4.3)
and that the Total Energy of a System is

E = KE + PE (1.4.4)

The Kinetic Energy is

1
KE = mv2
2
1 dx
= m
2 dt
1
= mA2 ω2 cos2 (ωt + δ) (1.4.5)
2

David S. Latchman ©2009


Oscillatory Motion 5
The Potential Energy is
1
U = kx2
2
1
= kA2 sin2 (ωt + δ) (1.4.6)
2
Adding eq. (1.4.5) and eq. (1.4.6) gives
1
E = kA2 (1.4.7)
2

1.4.4 Damped Harmonic Motion


dx
Fd = −bv = −b (1.4.8)

FT
dt
where b is the damping coefficient. The equation of motion for a damped oscillating
system becomes
dx d2 x
− kx − b = m 2 (1.4.9)
dt dt
Solving eq. (1.4.9) goves
x = Ae−αt sin (ω0 t + δ) (1.4.10)
We find that
RA
b
α= (1.4.11)
2m
r
k b2
ω0 = −
m 4m2
r
b2
= ω20 −
4m2
q
= ω20 − α2 (1.4.12)
D

1.4.5 Small Oscillations

The Energy of a system is


1
E = K + V(x) = mv(x)2 + V(x) (1.4.13)
2
We can solve for v(x), r
2
v(x) = (E − V(x)) (1.4.14)
m
where E ≥ V(x) Let the particle move in the potential valley, x1 ≤ x ≤ x2 , the potential
can be approximated by the Taylor Expansion
" # " 2 #
dV(x) 1 2 d V(x)
V(x) = V(xe ) + (x − xe ) + (x − xe ) + ··· (1.4.15)
dx x=xe 2 dx2 x=xe

©2009 David S. Latchman


6 Classical Mechanics
At the points of inflection, the derivative dV/dx is zero and d V/dx2 is positive. This
2

means that the potential energy for small oscillations becomes

1
V(x) u V(xe ) + k(x − xe )2 (1.4.16)
2
where " #
d2 V(x)
k≡ ≥0 (1.4.17)
dx2 x=xe

As V(xe ) is constant, it has no consequences to physical motion and can be dropped.


We see that eq. (1.4.16) is that of simple harmonic motion.

1.4.6 Coupled Harmonic Oscillators

FT
Consider the case of a simple pendulum of length, `, and the mass of the bob is m1 .
For small displacements, the equation of motion is

θ̈ + ω0 θ = 0

We can express this in cartesian coordinates, x and y, where


(1.4.18)
RA
x = ` cos θ ≈ ` (1.4.19)
y = ` sin θ ≈ `θ (1.4.20)

eq. (1.4.18) becomes


ÿ + ω0 y = 0 (1.4.21)
This is the equivalent to the mass-spring system where the spring constant is
mg
k = mω20 = (1.4.22)
`
D

This allows us to to create an equivalent three spring system to our coupled pendulum
system. The equations of motion can be derived from the Lagrangian, where

L=T−V
1 2 1 2 1 2 1 2 1 2
 
= m ẏ1 + m ẏ2 − ky1 + κ y2 − y1 + ky2
2 2 2 2 2
1  2  1   2 
= m y˙1 + y˙2 2 − k y21 + y22 + κ y2 − y1 (1.4.23)
2 2
We can find the equations of motion of our system

d ∂L ∂L
!
= (1.4.24)
dt ∂ ẏn ∂yn
1
Add figure with coupled pendulum-spring system

David S. Latchman ©2009


Oscillatory Motion 7
The equations of motion are
m ÿ1 = −ky1 + κ y2 − y1

(1.4.25)
m ÿ2 = −ky2 + κ y2 − y1

(1.4.26)
We assume solutions for the equations of motion to be of the form
y1 = cos(ωt + δ1 ) y2 = B cos(ωt + δ2 )
(1.4.27)
ÿ1 = −ωy1 ÿ2 = −ωy2
Substituting the values for ÿ1 and ÿ2 into the equations of motion yields
 
k + κ − mω2 y1 − κy2 = 0 (1.4.28)
 
−κy1 + k + κ − mω2 y2 = 0 (1.4.29)
We can get solutions from solving the determinant of the matrix

FT
k + κ − mω2

−κ  = 0 (1.4.30)
−κ k + κ − mω2
Solving the determinant gives
 2  
mω2 − 2mω2 (k + κ) + k2 + 2kκ = 0 (1.4.31)
This yields
g

k
=


`

ω2 =  m
RA

(1.4.32)

 k + 2κ g 2κ
= +


m ` m

We can now determine exactly how the masses move with each mode by substituting
ω2 into the equations of motion. Where
k
ω2 = We see that
m
k + κ − mω2 = κ (1.4.33)
Substituting this into the equation of motion yields
y1 = y2 (1.4.34)
D

We see that the masses move in phase with each other. You will also notice
the absense of the spring constant term, κ, for the connecting spring. As the
masses are moving in step, the spring isn’t stretching or compressing and hence
its absence in our result.
k+κ
ω2 = We see that
m
k + κ − mω2 = −κ (1.4.35)
Substituting this into the equation of motion yields
y1 = −y2 (1.4.36)
Here the masses move out of phase with each other. In this case we see the
presence of the spring constant, κ, which is expected as the spring playes a role.
It is being stretched and compressed as our masses oscillate.

©2009 David S. Latchman


8 Classical Mechanics
1.4.7 Doppler Effect

The Doppler Effect is the shift in frequency and wavelength of waves that results from
a source moving with respect to the medium, a receiver moving with respect to the
medium or a moving medium.

Moving Source If a source is moving towards an observer, then in one period, τ0 , it


moves a distance of vs τ0 = vs / f0 . The wavelength is decreased by
vs v − vs
λ0 = λ − − (1.4.37)
f0 f0
The frequency change is
v v
 
f0 = = f0 (1.4.38)
λ0 v − vs

FT
Moving Observer As the observer moves, he will measure the same wavelength, λ, as
if at rest but will see the wave crests pass by more quickly. The observer measures
a modified wave speed.
v0 = v + |vr | (1.4.39)
The modified frequency becomes
v0 vr
 
f0 = = f0 1 + (1.4.40)
λ
RA
v

Moving Source and Moving Observer We can combine the above two equations
v − vs
λ0 = (1.4.41)
f0
v = v − vr
0
(1.4.42)

To give a modified frequency of


v0 v − vr
 
f = 0 =
0
f0 (1.4.43)
λ v − vs
D

1.5 Rotational Motion about a Fixed Axis

1.5.1 Moment of Inertia


Z
I= R2 dm (1.5.1)

1.5.2 Rotational Kinetic Energy


1
K = Iω2 (1.5.2)
2

David S. Latchman ©2009


Rotational Motion about a Fixed Axis 9
1.5.3 Parallel Axis Theorem

I = Icm + Md2 (1.5.3)

1.5.4 Torque

τ=r×F (1.5.4)
τ = Iα (1.5.5)

where α is the angular acceleration.

1.5.5 Angular Momentum

we can find the Torque FT


L = Iω

dL
(1.5.6)
RA
τ= (1.5.7)
dt

1.5.6 Kinetic Energy in Rolling

With respect to the point of contact, the motion of the wheel is a rotation about the
point of contact. Thus
1
K = Krot = Icontact ω2 (1.5.8)
2
D

Icontact can be found from the Parallel Axis Theorem.

Icontact = Icm + MR2 (1.5.9)

Substitute eq. (1.5.8) and we have

1 
K= Icm + MR2 ω2
2
1 1
= Icm ω2 + mv2 (1.5.10)
2 2

The kinetic energy of an object rolling without slipping is the sum of hte kinetic energy
of rotation about its center of mass and the kinetic energy of the linear motion of the
object.

©2009 David S. Latchman


10 Classical Mechanics
1.6 Dynamics of Systems of Particles

1.6.1 Center of Mass of a System of Particles

Position Vector of a System of Particles

m1 r1 + m2 r2 + m3 r3 + · · · + mN rN
R= (1.6.1)
M

Velocity Vector of a System of Particles

dR
V=

FT
dt
m1 v1 + m2 v2 + m3 v3 + · · · + mN vN
= (1.6.2)
M

Acceleration Vector of a System of Particles

dV
A=
dt
RA
m1 a1 + m2 a2 + m3 a3 + · · · + mN aN
= (1.6.3)
M

1.7 Central Forces and Celestial Mechanics

1.7.1 Newton’s Law of Universal Gravitation


GMm
 
F=− r̂ (1.7.1)
r2
D

1.7.2 Potential Energy of a Gravitational Force


GMm
U(r) = − (1.7.2)
r

1.7.3 Escape Speed and Orbits

The energy of an orbiting body is


E=T+U
1 GMm
= mv2 − (1.7.3)
2 r

David S. Latchman ©2009


Central Forces and Celestial Mechanics 11
The escape speed becomes
1 GMm
E = mv2esc − =0 (1.7.4)
2 RE
Solving for vesc we find
r
2GM
vesc = (1.7.5)
Re

1.7.4 Kepler’s Laws

First Law The orbit of every planet is an ellipse with the sun at a focus.

Second Law A line joining a planet and the sun sweeps out equal areas during equal

FT
intervals of time.

Third Law The square of the orbital period of a planet is directly proportional to the
cube of the semi-major axis of its orbit.

T2
=C (1.7.6)
R3
where C is a constant whose value is the same for all planets.
RA
1.7.5 Types of Orbits

The Energy of an Orbiting Body is defined in eq. (1.7.3), we can classify orbits by their
eccentricities.

Circular Orbit A circular orbit occurs when there is an eccentricity of 0 and the orbital
energy is less than 0. Thus
D

1 2 GM
v − =E<0 (1.7.7)
2 r
The Orbital Velocity is
r
GM
v= (1.7.8)
r

Elliptic Orbit An elliptic orbit occurs when the eccentricity is between 0 and 1 but the
specific energy is negative, so the object remains bound.
r
2 1
 
v= GM − (1.7.9)
r a

where a is the semi-major axis

©2009 David S. Latchman


12 Classical Mechanics
Parabolic Orbit A Parabolic Orbit occurs when the eccentricity is equal to 1 and the
orbital velocity is the escape velocity. This orbit is not bounded. Thus

1 2 GM
v − =E=0 (1.7.10)
2 r
The Orbital Velocity is
r
2GM
v = vesc = (1.7.11)
r

Hyperbolic Orbit In the Hyperbolic Orbit, the eccentricity is greater than 1 with an
orbital velocity in excess of the escape velocity. This orbit is also not bounded.
r
GM
v∞ =

FT
(1.7.12)
a

1.7.6 Derivation of Vis-viva Equation

The total energy of a satellite is

1 GMm
E = mv2 − (1.7.13)
RA
2 r
For an elliptical or circular orbit, the specific energy is

GMm
E=− (1.7.14)
2a
Equating we get
2 1
 
v = GM −
2
(1.7.15)
r a
D

1.8 Three Dimensional Particle Dynamics

1.9 Fluid Dynamics

When an object is fully or partially immersed, the buoyant force is equal to the weight
of fluid displaced.

1.9.1 Equation of Continuity

ρ1 v1 A1 = ρ2 v2 A2 (1.9.1)

David S. Latchman ©2009


Non-inertial Reference Frames 13
1.9.2 Bernoulli’s Equation
1
P + ρv2 + ρgh = a constant (1.9.2)
2

1.10 Non-inertial Reference Frames

1.11 Hamiltonian and Lagrangian Formalism

1.11.1 Lagrange’s Function (L)

L=T−V (1.11.1)

FT
where T is the Kinetic Energy and V is the Potential Energy in terms of Generalized
Coordinates.

1.11.2 Equations of Motion(Euler-Lagrange Equation)

∂L d ∂L
!
= (1.11.2)
RA
∂q dt ∂q̇

1.11.3 Hamiltonian

H =T+V
= pq̇ − L(q, q̇) (1.11.3)

where
D

∂H
= q̇ (1.11.4)
∂p
∂H ∂L
=−
∂q ∂x
= −ṗ (1.11.5)

©2009 David S. Latchman


14 Classical Mechanics

FT
RA
D

David S. Latchman ©2009


Chapter 2
Electromagnetism

2.1

2.1.1
Electrostatics

Coulomb’s Law
FT
The force between two charged particles, q1 and q2 is defined by Coulomb’s Law.
RA
!
1 q1 q2
F12 = r̂12 (2.1.1)
4π0 r212

where 0 is the permitivitty of free space, where

0 = 8.85 × 10−12 C2 N.m2 (2.1.2)

2.1.2 Electric Field of a point charge


D

The electric field is defined by mesuring the magnitide and direction of an electric
force, F, acting on a test charge, q0 .
F
E≡ (2.1.3)
q0
The Electric Field of a point charge, q is

1 q
E= r̂ (2.1.4)
4π0 r2

In the case of multiple point charges, qi , the electric field becomes

n
1 X qi
E(r) = r̂i (2.1.5)
4π0 i=1 r2i
16 Electromagnetism
Electric Fields and Continuous Charge Distributions

If a source is distributed continuously along a region of space, eq. (2.1.5) becomes


Z
1 1
E(r) = r̂dq (2.1.6)
4π0 r2
If the charge was distributed along a line with linear charge density, λ,
dq
λ= (2.1.7)
dx
The Electric Field of a line charge becomes

λ
Z
1
E(r) = r̂dx (2.1.8)
4π0 r2

FT
line

In the case where the charge is distributed along a surface, the surface charge density
is, σ
Q dq
σ= = (2.1.9)
A dA
The electric field along the surface becomes

σ
Z
1
E(r) = r̂dA (2.1.10)
RA
4π0 r2
Surface

In the case where the charge is distributed throughout a volume, V, the volume charge
density is
Q dq
ρ= = (2.1.11)
V dV
The Electric Field is
ρ
Z
1
E(r) = r̂dV (2.1.12)
4π0 r2
Volume
D

2.1.3 Gauss’ Law

The electric field through a surface is


I I
Φ= dΦ = E · dA (2.1.13)
surface S surface S

The electric flux through a closed surface encloses a net charge.


I
Q
E · dA = (2.1.14)
0
where Q is the charge enclosed by our surface.

David S. Latchman ©2009


Electrostatics 17
2.1.4 Equivalence of Coulomb’s Law and Gauss’ Law

The total flux through a sphere is


I
q
E · dA = E(4πr2 ) = (2.1.15)
0
From the above, we see that the electric field is
q
E= (2.1.16)
4π0 r2

2.1.5 Electric Field due to a line of charge

Consider an infinite rod of constant charge density, λ. The flux through a Gaussian

FT
cylinder enclosing the line of charge is
Z Z Z
Φ= E · dA + E · dA + E · dA (2.1.17)
top surface bottom surface side surface

At the top and bottom surfaces, the electric field is perpendicular to the area vector, so
for the top and bottom surfaces,
E · dA = 0 (2.1.18)
RA
At the side, the electric field is parallel to the area vector, thus

E · dA = EdA (2.1.19)

Thus the flux becomes, Z Z


Φ= E · dA = E dA (2.1.20)
side sirface

The area in this case is the surface area of the side of the cylinder, 2πrh.
D

Φ = 2πrhE (2.1.21)

Applying Gauss’ Law, we see that Φ = q/0 . The electric field becomes

λ
E= (2.1.22)
2π0 r

2.1.6 Electric Field in a Solid Non-Conducting Sphere

Within our non-conducting sphere or radius, R, we will assume that the total charge,
Q is evenly distributed throughout the sphere’s volume. So the charge density of our
sphere is
Q Q
ρ= = 4 (2.1.23)
V 3
πR 3

©2009 David S. Latchman


18 Electromagnetism
The Electric Field due to a charge Q is
Q
E= (2.1.24)
4π0 r2
As the charge is evenly distributed throughout the sphere’s volume we can say that
the charge density is
dq = ρdV (2.1.25)
where dV = 4πr2 dr. We can use this to determine the field inside the sphere by
summing the effect of infinitesimally thin spherical shells
Z E Z r
dq
E= dE = 2
0 0 4πr
ρ
Z r
= dr
0 0

FT
Qr
= 4 (2.1.26)
3
π 0 R3

2.1.7 Electric Potential Energy


1
U(r) = qq0 r (2.1.27)
4π0
RA
2.1.8 Electric Potential of a Point Charge

The electrical potential is the potential energy per unit charge that is associated with a
static electrical field. It can be expressed thus
U(r) = qV(r) (2.1.28)
And we can see that
1 q
V(r) = (2.1.29)
D

4π0 r
A more proper definition that includes the electric field, E would be
Z
V(r) = − E · d` (2.1.30)
C

where C is any path, starting at a chosen point of zero potential to our desired point.
The difference between two potentials can be expressed such
Z b Z a
V(b) − V(a) = − E · d` + E · d`
Z b
=− E · d` (2.1.31)
a

David S. Latchman ©2009


Electrostatics 19
This can be further expressed
Z b
V(b) − V(a) = (∇V) · d` (2.1.32)
a

And we can show that


E = −∇V (2.1.33)

2.1.9 Electric Potential due to a line charge along axis

Let us consider a rod of length, `, with linear charge density, λ. The Electrical Potential
due to a continuous distribution is

FT
Z Z
1 dq
V= dV = (2.1.34)
4π0 r

The charge density is


dq = λdx (2.1.35)
Substituting this into the above equation, we get the electrical potential at some distance
x along the rod’s axis, with the origin at the start of the rod.
RA
1 dq
dV =
4π0 x
1 λdx
= (2.1.36)
4π0 x
This becomes
λ x2
 
V= ln (2.1.37)
4π0 x1
where x1 and x2 are the distances from O, the end of the rod.
Now consider that we are some distance, y, from the axis of the rod of length, `. We
D

again look at eq. (2.1.34), where r is the distance of the point P from the rod’s axis.
Z
1 dq
V=
4π0 r
Z `
1 λdx
=
4π0 0 x2 + y2  12
λ
   12 `
= ln x + x2 + y2
4π0 0
λ  12 
 
= ln ` + `2 + y2 − ln y
4π0
 1 
λ  ` + `2 + y2 2 
= ln   (2.1.38)
4π0  d


©2009 David S. Latchman


20 Electromagnetism
2.2 Currents and DC Circuits

2.3 Magnetic Fields in Free Space

2.4 Lorentz Force

FT
4

2.5 Induction

5
RA
2.6 Maxwell’s Equations and their Applications

2.7 Electromagnetic Waves


D

2.8 AC Circuits

2.9 Magnetic and Electric Fields in Matter

David S. Latchman ©2009


Capacitance 21
2.10 Capacitance
Q = CV (2.10.1)

2.11 Energy in a Capacitor

Q2
U=
2C
CV 2
=
2
QV

FT
= (2.11.1)
2

2.12 Energy in an Electric Field

U 0 E2
u≡ = (2.12.1)
volume 2
RA
2.13 Current
dQ
I≡ (2.13.1)
dt

2.14 Current Destiny


D

Z
I= J · dA (2.14.1)
A

2.15 Current Density of Moving Charges


I
J= = ne qvd (2.15.1)
A

2.16 Resistance and Ohm’s Law


V
R≡ (2.16.1)
I

©2009 David S. Latchman


22 Electromagnetism
2.17 Resistivity and Conductivity

L
R=ρ (2.17.1)
A

E = ρJ (2.17.2)

J = σE (2.17.3)

2.18 Power

2.19 Kirchoff’s Loop Rules

Write Here
FT
P = VI (2.18.1)
RA
2.20 Kirchoff’s Junction Rule

Write Here

2.21 RC Circuits
D

Q
E − IR − =0 (2.21.1)
C

2.22 Maxwell’s Equations

2.22.1 Integral Form

Gauss’ Law for Electric Fields


w Q
E · dA = (2.22.1)
0
closed surface

David S. Latchman ©2009


Speed of Propagation of a Light Wave 23
Gauss’ Law for Magnetic Fields
w
B · dA = 0 (2.22.2)
closed surface

Ampère’s Law
z d w
B · ds = µ0 I + µ0 0 E · dA (2.22.3)
dt
surface

Faraday’s Law
z d w
E · ds = − B · dA (2.22.4)
dt
surface

2.22.2 Differential Form


Gauss’ Law for Electric Fields

Gauss’ Law for Magnetism

Ampère’s Law
FT
∇·E=

∇·B=0
ρ
0
(2.22.5)

(2.22.6)
RA
∂E
∇ × B = µ0 J + µ0 0 (2.22.7)
∂t
Faraday’s Law
∂B
∇·E=− (2.22.8)
∂t

2.23 Speed of Propagation of a Light Wave


1
D

c= √ (2.23.1)
µ0 0
In a material with dielectric constant, κ,
√ c
c κ = (2.23.2)
n
where n is the refractive index.

2.24 Relationship between E and B Fields

E = cB (2.24.1)
E·B=0 (2.24.2)

©2009 David S. Latchman


24 Electromagnetism
2.25 Energy Density of an EM wave
!
1 B2
u= + 0 E2 (2.25.1)
2 µ0

2.26 Poynting’s Vector


1
S= E×B (2.26.1)
µ0

FT
RA
D

David S. Latchman ©2009


Chapter 3
Optics & Wave Phonomena

3.1
1

3.2
Wave Properties

Superposition
FT
RA
2

3.3 Interference
3
D

3.4 Diffraction
4

3.5 Geometrical Optics


5

3.6 Polarization
6
26 Optics & Wave Phonomena
3.7 Doppler Effect
7

3.8 Snell’s Law

3.8.1 Snell’s Law

n1 sin θ1 = n2 sin θ2 (3.8.1)

3.8.2 Critical Angle and Snell’s Law

FT
The critical angle, θc , for the boundary seperating two optical media is the smallest
angle of incidence, in the medium of greater index, for which light is totally refelected.
From eq. (3.8.1), θ1 = 90 and θ2 = θc and n2 > n1 .

n1 sin 90 = n2 sinθc
sin θc =
n1
(3.8.2)
RA
n2
D

David S. Latchman ©2009


Chapter 4
Thermodynamics & Statistical Mechanics

4.1
1

4.2
FT
Laws of Thermodynamics

Thermodynamic Processes
RA
2

4.3 Equations of State


3
D

4.4 Ideal Gases


4

4.5 Kinetic Theory


5

4.6 Ensembles
6
28 Thermodynamics & Statistical Mechanics
4.7 Statistical Concepts and Calculation of Thermody-
namic Properties

4.8 Thermal Expansion & Heat Transfer

FT
4.9 Heat Capacity
 
Q = C T f − Ti (4.9.1)

where C is the Heat Capacity and T f and Ti are the final and initial temperatures
respectively.
RA
4.10 Specific Heat Capacity
 
Q = cm T f − ti (4.10.1)

where c is the specific heat capacity and m is the mass.


D

4.11 Heat and Work


Z Vf
W= PdV (4.11.1)
Vi

4.12 First Law of Thermodynamics

dEint = dQ − dW (4.12.1)

where dEint is the internal energy of the system, dQ is the Energy added to the system
and dW is the work done by the system.

David S. Latchman ©2009


Work done by Ideal Gas at Constant Temperature 29
4.12.1 Special Cases to the First Law of Thermodynamics
Adiabatic Process During an adiabatic process, the system is insulated such that there
is no heat transfer between the system and its environment. Thus dQ = 0, so

∆Eint = −W (4.12.2)

If work is done on the system, negative W, then there is an increase in its internal
energy. Conversely, if work is done by the system, positive W, there is a decrease
in the internal energy of the system.

Constant Volume (Isochoric) Process If the volume is held constant, then the system
can do no work, δW = 0, thus
∆Eint = Q (4.12.3)

FT
If heat is added to the system, the temperature increases. Conversely, if heat is
removed from the system the temperature decreases.

Closed Cycle In this situation, after certain interchanges of heat and work, the system
comes back to its initial state. So ∆Eint remains the same, thus

∆Q = ∆W (4.12.4)

The work done by the system is equal to the heat or energy put into it.
RA
Free Expansion In this process, no work is done on or by the system. Thus ∆Q =
∆W = 0,
∆Eint = 0 (4.12.5)

4.13 Work done by Ideal Gas at Constant Temperature


Starting with eq. (4.11.1), we substitute the Ideal gas Law, eq. (4.15.1), to get
D

Z Vf
dV
W = nRT
Vi V
Vf
= nRT ln (4.13.1)
Vi

4.14 Heat Conduction Equation


The rate of heat transferred, H, is given by

Q TH − TC
H= = kA (4.14.1)
t L
where k is the thermal conductivity.

©2009 David S. Latchman


30 Thermodynamics & Statistical Mechanics
4.15 Ideal Gas Law
PV = nRT (4.15.1)
where
n = Number of moles
P = Pressure
V = Volume
T = Temperature
and R is the Universal Gas Constant, such that
R ≈ 8.314 J/mol. K

FT
We can rewrite the Ideal gas Law to say
PV = NkT (4.15.2)
where k is the Boltzmann’s Constant, such that
R
k= ≈ 1.381 × 10−23 J/K
NA
RA
4.16 Stefan-Boltzmann’s FormulaStefan-Boltzmann’s Equa-
tion
P(T) = σT4 (4.16.1)

4.17 RMS Speed of an Ideal Gas


D

r
3RT
vrms = (4.17.1)
M

4.18 Translational Kinetic Energy


3
K̄ = kT (4.18.1)
2

4.19 Internal Energy of a Monatomic gas


3
Eint = nRT (4.19.1)
2

David S. Latchman ©2009


Molar Specific Heat at Constant Volume 31
4.20 Molar Specific Heat at Constant Volume

Let us define, CV such that


Q = nCV ∆T (4.20.1)

Substituting into the First Law of Thermodynamics, we have

∆Eint + W = nCV ∆T (4.20.2)

At constant volume, W = 0, and we get

1 ∆Eint

FT
CV = (4.20.3)
n ∆T

Substituting eq. (4.19.1), we get

3
CV = R = 12.5 J/mol.K (4.20.4)
2
RA
4.21 Molar Specific Heat at Constant Pressure

Starting with
Q = nCp ∆T (4.21.1)

and
D

∆Eint = Q − W
⇒ nCV ∆T = nCp ∆T + nR∆T
∴ CV = Cp − R (4.21.2)

4.22 Equipartition of Energy

!
f
CV = R = 4.16 f J/mol.K (4.22.1)
2

where f is the number of degrees of freedom.

©2009 David S. Latchman


Thermodynamics & Statistical Mechanics

©2009
FT Degrees of Freedom Predicted Molar Specific Heats
CP = CV + R
RA
Molecule Translational Rotational Vibrational Total ( f ) CV
3 5
Monatomic 3 0 0 3 2
R 2
R
5 7
Diatomic 3 2 2 5 2
R 2
R
Polyatomic (Linear) 3 3 3n − 5 6 3R 4R
Polyatomic (Non-Linear) 3 3 3n − 6 6 3R 4R
Table 4.22.1: Table of Molar Specific Heats
D

David S. Latchman
32
Adiabatic Expansion of an Ideal Gas 33
4.23 Adiabatic Expansion of an Ideal Gas
PV γ = a constant (4.23.1)
where γ = CCVP .
We can also write
TV γ−1 = a constant (4.23.2)

4.24 Second Law of Thermodynamics


Something.

FT
RA
D

©2009 David S. Latchman


34 Thermodynamics & Statistical Mechanics

FT
RA
D

David S. Latchman ©2009


Chapter 5
Quantum Mechanics

5.1
1

5.2
Fundamental Concepts

Schrödinger Equation
FT
RA
Let us define Ψ to be
Ψ = Ae−iω(t− v )
x
(5.2.1)
Simplifying in terms of Energy, E, and momentum, p, we get
i(Et−px)
Ψ = Ae− ~ (5.2.2)

We obtain Schrödinger’s Equation from the Hamiltonian


D

H =T+V (5.2.3)

To determine E and p,

∂2 Ψ p2
= − Ψ (5.2.4)
∂x2 ~2
∂Ψ iE
= Ψ (5.2.5)
∂t ~
and
p2
H= +V (5.2.6)
2m
This becomes

EΨ = HΨ (5.2.7)
36 Quantum Mechanics
~ ∂Ψ ∂Ψ 2
EΨ = − p2 Ψ = −~2 2
i ∂t ∂x
The Time Dependent Schrödinger’s Equation is

∂Ψ ~ 2 ∂2 Ψ
i~ =− + V(x)Ψ (5.2.8)
∂t 2m ∂x2
The Time Independent Schrödinger’s Equation is

~ 2 ∂2 Ψ
EΨ = − + V(x)Ψ (5.2.9)
2m ∂x2

5.2.1 Infinite Square Wells

FT
Let us consider a particle trapped in an infinite potential well of size a, such that

for 0 < x < a


(
0
V(x) =
∞ for |x| > a,

so that a nonvanishing force acts only at ±a/2. An energy, E, is assigned to the system
such that the kinetic energy of the particle is E. Classically, any motion is forbidden
outside of the well because the infinite value of V exceeds any possible choice of E.
RA
Recalling the Schrödinger Time Independent Equation, eq. (5.2.9), we substitute V(x)
and in the region (−a/2, a/2), we get

~2 d2 ψ
− = Eψ (5.2.10)
2m dx2
This differential is of the form
d2 ψ
2
+ k2 ψ = 0 (5.2.11)
dx
where
D

r
2mE
k= (5.2.12)
~2
We recognize that possible solutions will be of the form

cos kx and sin kx

As the particle is confined in the region 0 < x < a, we say

A cos kx + B sin kx for 0 < x < a


(
ψ(x) =
0 for |x| > a

We have known boundary conditions for our square well.

ψ(0) = ψ(a) = 0 (5.2.13)

David S. Latchman ©2009


Schrödinger Equation 37
It shows that

⇒ A cos 0 + B sin 0 = 0
∴A=0 (5.2.14)

We are now left with

B sin ka = 0
ka = 0; π; 2π; 3π; · · ·
(5.2.15)

While mathematically, n can be zero, that would mean there would be no wave function,
so we ignore this result and say

FT

kn = for n = 1, 2, 3, · · ·
a
Substituting this result into eq. (5.2.12) gives

nπ 2mEn
kn = = (5.2.16)
a ~
Solving for En gives
RA
n2 π2 ~2
En = (5.2.17)
2ma2
We cna now solve for B by normalizing the function
Z a
a
|B|2 sin2 kxdx = |A|2 = 1
0 2
2
So |A|2 = (5.2.18)
a
So we can write the wave function as
D

r
2 nπx
 
ψn (x) = sin (5.2.19)
a a

5.2.2 Harmonic Oscillators

Classically, the harmonic oscillator has a potential energy of

1
V(x) = kx2 (5.2.20)
2
So the force experienced by this particle is

dV
F=− = −kx (5.2.21)
dx

©2009 David S. Latchman


38 Quantum Mechanics
where k is the spring constant. The equation of motion can be summed us as

d2 x
m 2 = −kx (5.2.22)
dt
And the solution of this equation is
 
x(t) = A cos ω0 t + φ (5.2.23)

where the angular frequency, ω0 is


r
k
ω0 = (5.2.24)
m
The Quantum Mechanical description on the harmonic oscillator is based on the eigen-

FT
function solutions of the time-independent Schrödinger’s equation. By taking V(x)
from eq. (5.2.20) we substitute into eq. (5.2.9) to get

d2 ψ 2m k 2
!
mk 2 2E
 
= x − E ψ = x − ψ
dx2 ~2 2 ~2 k

With some manipulation, we get


√
d2 ψ  mk 2 2E m 
r 
RA
~
√ 2
=  x −  ψ
mk dx ~ ~ k 

This step allows us to to keep some of constants out of the way, thus giving us

mk 2
ξ2 = x (5.2.25)
~r
2E m 2E
and λ = = (5.2.26)
~ k ~ω0
This leads to the more compact
D

d2 ψ  2 
= ξ − λ ψ (5.2.27)
dξ2
where the eigenfunction ψ will be a function of ξ. λ assumes an eigenvalue anaglaous
to E.
From eq. (5.2.25), we see that the maximum value can be determined to be

mk 2
ξmax =
2
A (5.2.28)
~
Using the classical connection between A and E, allows us to say

mk 2E
ξmax =
2
=λ (5.2.29)
~ k

David S. Latchman ©2009


Schrödinger Equation 39
From eq. (5.2.27), we see that in a quantum mechanical oscillator, there are non-
vanishing solutions in the forbidden regions, unlike in our classical case.
A solution to eq. (5.2.27) is
ψ(ξ) = e−ξ /2
2
(5.2.30)
where

= −ξe−ξ /2
2


dψ 2 −xi2 /2 −ξ2 /2
e−ξ /2
  2
and 2
= ξ e − e = ξ 2
− 1

This gives is a special solution for λ where

λ0 = 1 (5.2.31)

FT
Thus eq. (5.2.26) gives the energy eigenvalue to be
~ω0 ~ω0
E0 = λ0 = (5.2.32)
2 2
The eigenfunction e−ξ /2 corresponds to a normalized stationary-state wave function
2

! 18 √
mk mk x2 /2~ −iE0 t/~
Ψ0 (x, t) = 2 2 e−
RA
e (5.2.33)
π~
This solution of eq. (5.2.27) produces the smallest possibel result of λ and E. Hence,
Ψ0 and E0 represents the ground state of the oscillator. and the quantity ~ω0 /2 is the
zero-point energy of the system.

5.2.3 Finite Square Well

For the Finite Square Well, we have a potential region where


D

(
−V0 for −a ≤ x ≤ a
V(x) =
0 for |x| > a

We have three regions

Region I: x < −a In this region, The potential, V = 0, so Schrödinger’s Equation be-


comes
~2 d2 ψ
− = Eψ
2m dx2
d2 ψ
⇒ 2 = κ2 ψ
√ dx
−2mE
where κ=
~

©2009 David S. Latchman


40 Quantum Mechanics
This gives us solutions that are

ψ(x) = A exp(−κx) + B exp(κx)

As x → ∞, the exp(−κx) term goes to ∞; it blows up and is not a physically


realizable function. So we can drop it to get

ψ(x) = Beκx for x < −a (5.2.34)

Region II: −a < x < a In this region, our potential is V(x) = V0 . Substitutin this into
the Schrödinger’s Equation, eq. (5.2.9), gives

~2 d2 ψ
− − V0 ψ = Eψ
2m dx2

FT
d2 ψ
or 2
= −l2 ψ
p dx
2m (E + V0 )
where l ≡ (5.2.35)
~
We notice that E > −V0 , making l real and positive. Thus our general solution
becomes
ψ(x) = C sin(lx) + D cos(lx) for −a < x < a (5.2.36)
RA
Region III: x > a Again this Region is similar to Region III, where the potential, V = 0.
This leaves us with the general solution

ψ(x) = F exp(−κx) + G exp(κx)

As x → ∞, the second term goes to infinity and we get

ψ(x) = Fe−κx for x > a (5.2.37)

This gives us
D

 κx

 Be for x < a
ψ(x) =  for 0 < x < a

D cos(lx) (5.2.38)

for x > a

 Fe−κx

5.2.4 Hydrogenic Atoms

5.3 Spin
3

David S. Latchman ©2009


Angular Momentum 41
5.4 Angular Momentum
4

5.5 Wave Funtion Symmetry


5

5.6 Elementary Perturbation Theory

FT
6
RA
D

©2009 David S. Latchman


42 Quantum Mechanics

FT
RA
D

David S. Latchman ©2009


Chapter 6
Atomic Physics

6.1
1

6.2
Properties of Electrons

Bohr Model
FT
RA
To understand the Bohr Model of the Hydrogen atom, we will take advantage of our
knowlegde of the wavelike properties of matter. As we are building on a classical
model of the atom with a modern concept of matter, our derivation is considered to be
‘semi-classical’. In this model we have an electron of mass, me , and charge, −e, orbiting
a proton. The cetripetal force is equal to the Coulomb Force. Thus

1 e2 me v2
= (6.2.1)
4π0 r2 r
D

The Total Energy is the sum of the potential and kinetic energies, so

p2
E=K+U = − | f race2 4π0 r (6.2.2)
2me
We can further reduce this equation by subsituting the value of momentum, which we
find to be
p2 1 e2
= me v2 = (6.2.3)
2me 2 8π0 r
Substituting this into eq. (6.2.2), we get

e2 e2 e2
E= − =− (6.2.4)
8π0 r 4π0 r 8π0 r
At this point our classical description must end. An accelerated charged particle, like
one moving in circular motion, radiates energy. So our atome here will radiate energy
44 Atomic Physics
and our electron will spiral into the nucleus and disappear. To solve this conundrum,
Bohr made two assumptions.

1. The classical circular orbits are replaced by stationary states. These stationary
states take discreet values.

2. The energy of these stationary states are determined by their angular momentum
which must take on quantized values of ~.

L = n~ (6.2.5)

We can find the angular momentum of a circular orbit.

L = m3 vr (6.2.6)

FT
From eq. (6.2.1) we find v and by substitution, we find L.
r
m3 r
L=e (6.2.7)
4π0

Solving for r, gives


L2
r= (6.2.8)
me e2 /4π0
RA
We apply the condition from eq. (6.2.5)

n2 ~2
rn = = n2 a0 (6.2.9)
me e2 /4π0

where a0 is the Bohr radius.


a0 = 0.53 × 10−10 m (6.2.10)
Having discreet values for the allowed radii means that we will also have discreet
values for energy. Replacing our value of rn into eq. (6.2.4), we get
D

!
me e2 13.6
En = − 2 = − 2 eV (6.2.11)
2n 4π0 ~ n

6.3 Energy Quantization


3

6.4 Atomic Structure


4

David S. Latchman ©2009


Atomic Spectra 45
6.5 Atomic Spectra

6.5.1 Rydberg’s Equation

1 1 1
 
= RH 02 − 2 (6.5.1)
λ n n
where RH is the Rydberg constant.
For the Balmer Series, n0 = 2, which determines the optical wavelengths. For n0 = 3, we
get the infrared or Paschen series. The fundamental n0 = 1 series falls in the ultraviolet
region and is known as the Lyman series.

FT
6.6 Selection Rules

6.7 Black Body Radiation


RA
6.7.1 Plank Formula

8π~ f3
u( f, T) = 3 h f /kT (6.7.1)
c e −1

6.7.2 Stefan-Boltzmann Formula

P(T) = σT4 (6.7.2)


D

6.7.3 Wein’s Displacement Law

λmax T = 2.9 × 10−3 m.K (6.7.3)

6.7.4 Classical and Quantum Aspects of the Plank Equation

Rayleigh’s Equation

8π f 2
u( f, T) = 3 kT (6.7.4)
c

©2009 David S. Latchman


46 Atomic Physics
We can get this equation from Plank’s Equation, eq. (6.7.1). This equation is a classical
one and does not contain Plank’s constant in it. For this case we will look at the
situation where h f < kT. In this case, we make the approximation

ex ' 1 + x (6.7.5)

Thus the demonimator in eq. (6.7.1) becomes

hf hf
eh f /kT − 1 ' 1 + −1= (6.7.6)
kT kT
Thus eq. (6.7.1) takes the approximate form

8πh 3 kT 8π f 2
u( f, T) ' f = 3 kT (6.7.7)
c3 hf c

quantum effects.

Quantum
FT
As we can see this equation is devoid of Plank’s constant and thus independent of

At large frequencies, where h f > kT, quantum effects become apparent. We can
RA
estimate that
eh f /kT − 1 ' eh f /kT (6.7.8)
Thus eq. (6.7.1) becomes
8πh 3 −h f /kT
u( f, T) ' f e (6.7.9)
c3

6.8 X-Rays
D

6.8.1 Bragg Condition

2d sin θ = mλ (6.8.1)
for constructive interference off parallel planes of a crystal with lattics spacing, d.

6.8.2 The Compton Effect

The Compton Effect deals with the scattering of monochromatic X-Rays by atomic
targets and the observation that the wavelength of the scattered X-ray is greater than
the incident radiation. The photon energy is given by

hc
E = hυ = (6.8.2)
λ

David S. Latchman ©2009


Atoms in Electric and Magnetic Fields 47
The photon has an associated momentum

E
= pc (6.8.3)
E hυ h
⇒p = = = (6.8.4)
c c λ
The Relativistic Energy for the electron is

E2 = p2 c2 + m2e c4 (6.8.5)

where
p − p0 = P (6.8.6)
Squaring eq. (6.8.6) gives
p2 − 2p · p0 + p02 = P2 (6.8.7)

FT
Recall that E = pc and E 0 = cp0 , we have

c2 p2 − 2c2 p · p0 + c2 p02 = c2 P2
E 2 − 2E E 0 cos θ + E 02 = E2 − m2e c4 (6.8.8)

Conservation of Energy leads to

E + me c2 = E 0 + E (6.8.9)
RA
Solving

E − E 0 = E − me c2
E 2 − 2E E 0 + E 0 = E2 − 2Eme c2 + m2e c4 (6.8.10)
2E E 0 − 2E E 0 cos θ = 2Eme c2 − 2m2e c4 (6.8.11)

Solving leads to
h
∆λ = λ0 − λ = (1 − cos θ) (6.8.12)
me c
D

where λc = h
me c
is the Compton Wavelength.

h
λc = = 2.427 × 10−12 m (6.8.13)
me c

6.9 Atoms in Electric and Magnetic Fields

6.9.1 The Cyclotron Frequency

A test charge, q, with velocity v enters a uniform magnetic field, B. The force acting on
the charge will be perpendicular to v such that

FB = qv × B (6.9.1)

©2009 David S. Latchman


48 Atomic Physics
or more simply FB = qvB. As this traces a circular path, from Newton’s Second Law,
we see that
mv2
FB = = qvB (6.9.2)
R
Solving for R, we get
mv
R= (6.9.3)
qB
We also see that
qB
f = (6.9.4)
2πm
The frequency is depends on the charge, q, the magnetic field strength, B and the mass
of the charged particle, m.

FT
6.9.2 Zeeman Effect

The Zeeman effect was the splitting of spectral lines in a static magnetic field. This is
similar to the Stark Effect which was the splitting in the presence in a magnetic field.
In the Zeeman experiment, a sodium flame was placed in a magnetic field and its
spectrum observed. In the presence of the field, a spectral line of frequency, υ0 was
split into three components, υ0 − δυ, υ0 and υ0 + δυ. A classical analysis of this effect
allows for the identification of the basic parameters of the interacting system.
RA
The application of a constant magnetic field, B, allows for a direction in space in which
the electron motion can be referred. The motion of an electron can be attributed to a
simple harmonic motion under a binding force −kr, where the frequency is
r
1 k
υ0 = (6.9.5)
2π me
The magnetic field subjects the electron to an additional Lorentz Force, −ev × B. This
produces two different values for the angular velocity.
v = 2πrυ
D

The cetripetal force becomes


me v2
= 4π2 υ2 rme
r
Thus the certipetal force is
4π2 υ2 rme = 2πυreB + kr for clockwise motion
4π2 υ2 rme = −2πυreB + kr for counterclockwise motion
We use eq. (6.9.5), to emiminate k, to get
eB
υ2 − υ − υ0 = 0 (Clockwise)
2πme
eB
υ2 + υ − υ0 = 0 (Counterclockwise)
2πme

David S. Latchman ©2009


Atoms in Electric and Magnetic Fields 49
As we have assumed a small Lorentz force, we can say that the linear terms in υ are
small comapred to υ0 . Solving the above quadratic equations leads to

eB
υ = υ0 + for clockwise motion (6.9.6)
4πme
eB
υ = υ0 − for counterclockwise motion (6.9.7)
4πme
We note that the frequency shift is of the form

eB
δυ = (6.9.8)
4πme
If we view the source along the direction of B, we will observe the light to have two
polarizations, a closckwise circular polarization of υ0 + δυ and a counterclosckwise

FT
circular polarization of υ0 − δυ.

6.9.3 Franck-Hertz Experiment

The Franck-Hertz experiment, performed in 1914 by J. Franck and G. L. Hertz, mea-


sured the colisional excitation of atoms. Their experiement studied the current of
electrons in a tub of mercury vapour which revealed an abrupt change in the current
RA
at certain critical values of the applied voltage.1 They interpreted this observation as
evidence of a threshold for inelastic scattering in the colissions of electrons in mer-
cury atoms.The bahavior of the current was an indication that electrons could lose
a discreet amount of energy and excite mercury atoms in their passage through the
mercury vapour. These observations constituted a direct and decisive confirmation of
the existence os quantized energy levels in atoms.
D

1
Put drawing of Franck-Hertz Setup

©2009 David S. Latchman


50 Atomic Physics

FT
RA
D

David S. Latchman ©2009


Chapter 7
Special Relativity

7.1

7.1.1
Introductory Concepts

Postulates of Special Relativity


FT
1. The laws of Physics are the same in all inertial frames.
RA
2. The speed of light is the same in all inertial frames.

We can define
1
γ= q (7.1.1)
u2
1− c2

7.2 Time Dilation


D

∆t = γ∆t0 (7.2.1)
where ∆t0 is the time measured at rest relative to the observer, ∆t is the time measured
in motion relative to the observer.

7.3 Length Contraction

L0
L= (7.3.1)
γ
where L0 is the length of an object observed at rest relative to the observer and L is the
length of the object moving at a speed u relative to the observer.
52 Special Relativity
7.4 Simultaneity

7.5 Energy and Momentum

7.5.1 Relativistic Momentum & Energy

In relativistic mechanics, to be conserved, momentum and energy are defined as

FT
Relativistic Momentum
p̄ = γmv̄ (7.5.1)

Relativistic Energy
E = γmc2 (7.5.2)
RA
7.5.2 Lorentz Transformations (Momentum & Energy)

E
 
p0x= γ px − β (7.5.3)
c
py = py
0
(7.5.4)
= pz
p0z (7.5.5)
D

0
E E
 
=γ − βpx (7.5.6)
c c

7.5.3 Relativistic Kinetic Energy

K = E − mc2 (7.5.7)
 
 
 1 
= mc2  q − 1 (7.5.8)
v2
1−
 
c2

= mc2 γ − 1

(7.5.9)

David S. Latchman ©2009


Four-Vectors and Lorentz Transformation 53
7.5.4 Relativistic Dynamics (Collisions)

∆E
 
∆P0x = γ ∆Px − β (7.5.10)
c
∆P y = ∆P y
0
(7.5.11)
∆P0z
= ∆Pz (7.5.12)
∆E0
∆E
 
=γ − β∆Px (7.5.13)
c c

7.6 Four-Vectors and Lorentz Transformation

FT
We can represent an event in S with the column matrix, s,

 x 
 
 y 
s =   (7.6.1)
 z 

ict

A different Lorents frame, S0 , corresponds to another set of space time axes so that
 0 
RA
 x 
 y0 
s0 =  0  (7.6.2)
 
 z 
 0 
ict

The Lorentz Transformation is related by the matrix

 x   γ
 0  
0 0 iγβ   x 
 
 y0   0 1 0 0   y 
 0  =  (7.6.3)
     
 z   0 0 1 0   z 
  
−iγβ 0 0 γ
 0    
ict ict
D

We can express the equation in the form

s0 = L s (7.6.4)

The matrix L contains all the information needed to relate position four–vectors for
any given event as observed in the two Lorentz frames S and S0 . If we evaluate

 x 
 
h i  y 
s s=
T
x y z ict  z  = x + y + z − c t
  2 2 2 2 2
(7.6.5)
 
ict

Similarly we can show that

s0T s0 = x02 + y02 + z02 − c2 t02 (7.6.6)

©2009 David S. Latchman


54 Special Relativity
We can take any collection of four physical quantities to be four vector provided that
they transform to another Lorentz frame. Thus we have

 bx 
 
 b 
b =  y  (7.6.7)
 
 bz 
 
ibt

this can be transformed into a set of quantities of b0 in another frame S0 such that it
satisfies the transformation
b0 = L b (7.6.8)
Looking at the momentum-Energy four vector, we have

 px
 


FT
 p 
p =  y (7.6.9)
 
 pz


 
iE/c

Applying the same transformation rule, we have

p0 = L p (7.6.10)

We can also get a Lorentz-invariation relation between momentum and energy such
RA
that
p0T p0 = pT p (7.6.11)
The resulting equality gives

E02 E2
x + p y + pz −
p02 = + +
02 02 2 2 2
px p y p z − (7.6.12)
c2 c2

7.7 Velocity Addition


D

v−u
v0 = (7.7.1)
1 − uv
c2

7.8 Relativistic Doppler Formula


r r
c+u c−u
ῡ = υ0 let r = (7.8.1)
c−u c+u

We have

ῡreceding = rυ0 red-shift (Source Receding) (7.8.2)


υ0
ῡapproaching = blue-shift (Source Approaching) (7.8.3)
r

David S. Latchman ©2009


Lorentz Transformations 55
7.9 Lorentz Transformations
Given two reference frames S(x, y, z, t) and S0 (x0 , y0 , z0 , t0 ), where the S0 -frame is moving
in the x-direction, we have,

x0 = γ (x − ut) x = (x0 − ut0 ) (7.9.1)


y0 = y y = y0 (7.9.2)
z0 = y y0 = y (7.9.3)
u u 0
   
t = γ t − 2x
0
t = γ t + 2x
0
(7.9.4)
c c

7.10 Space-Time Interval

FT
(∆S)2 = (∆x)2 + ∆y 2 + (∆z)2 − c2 (∆t)2

(7.10.1)
Space-Time Intervals may be categorized into three types depending on their separa-
tion. They are

Time-like Interval

c2 ∆t2 > ∆r2 (7.10.2)


RA
∆S2 > 0 (7.10.3)

When two events are separated by a time-like interval, there is a cause-effect


relationship between the two events.

Light-like Interval

c2 ∆t2 = ∆r2 (7.10.4)


S =0
2
(7.10.5)
D

Space-like Intervals

c2 ∆t2 < ∆r2 (7.10.6)


∆S < 0 (7.10.7)

©2009 David S. Latchman


56 Special Relativity

FT
RA
D

David S. Latchman ©2009


Chapter 8
Laboratory Methods

8.1

8.1.1
Data and Error Analysis

Addition and Subtraction


FT
x=a+b−c (8.1.1)
RA
The Error in x is
(δx)2 = (δa)2 + (δb)2 + (δc)2 (8.1.2)

8.1.2 Multiplication and Division

a×b
x= (8.1.3)
c
D

The error in x is
!2
δx δa δb δc
 2  2  2
= + + (8.1.4)
x a b c

8.1.3 Exponent - (No Error in b)

x = ab (8.1.5)

The Error in x is
δx δa
 
=b (8.1.6)
x a
58 Laboratory Methods
8.1.4 Logarithms

Base e

x = ln a (8.1.7)
We find the error in x by taking the derivative on both sides, so

d ln a
δx = · δa
da
1
= · δa
a
δa
= (8.1.8)
a

Base 10

The Error in x can be derived as such

δx =
FT
x = log10 a

d(log a)
δa
(8.1.9)
RA
da
ln a
ln 10
= δa
da
1 δa
=
ln 10 a
δa
= 0.434 (8.1.10)
a

8.1.5 Antilogs
D

Base e

x = ea (8.1.11)
We take the natural log on both sides.

ln x = a ln e = a (8.1.12)

Applaying the same general method, we see

d ln x
δx = δa
dx
δx
⇒ = δa (8.1.13)
x

David S. Latchman ©2009


Instrumentation 59
Base 10

x = 10a (8.1.14)

We follow the same general procedure as above to get

log x = a log 10
log x
δx = δa
dx
1 d ln a
δx = δa
ln 10 dx
δx
= ln 10δa (8.1.15)
x

8.2

2
Instrumentation

FT
RA
8.3 Radiation Detection

8.4 Counting Statistics

Let’s assume that for a particular experiment, we are making countung measurements
D

for a radioactive source. In this experiment, we recored N counts in time T. The


counting rate for this trial is R = N/T. This rate should be close to the average
√ rate, R̄.
The standard deviation or the uncertainty of our count is a simply called the N rule.
So

σ= N (8.4.1)

Thus we can report our results as



Number of counts = N ± N (8.4.2)

We can find the count rate by dividing by T, so



N N
R= ± (8.4.3)
T T

©2009 David S. Latchman


60 Laboratory Methods
δN
The fractional uncertainty of our count is N
. We can relate this in terms of the count
rate.
δN
δR T δN
= N
=
R T
N

N
=
N
1
= (8.4.4)
N
We see that our uncertainty decreases as we take more counts, as to be expected.

8.5 Interaction of Charged Particles with Matter


5

8.6
6
Lasers and Optical Interferometers
FT
RA
8.7 Dimensional Analysis
Dimensional Analysis is used to understand physical situations involving a mis of
different types of physical quantities. The dimensions of a physical quantity are
associated with combinations of mass, length, time, electric charge, and temperature,
represented by symbols M, L, T, Q, and θ, respectively, each raised to rational powers.
D

8.8 Fundamental Applications of Probability and Statis-


tics
8

David S. Latchman ©2009


Chapter 9
GR0177 Exam Solutions

9.1 Acceleration of a Pendulum Bob

acent
FT θ
RA
atang

Figure 9.1.1: Acceleration components on pendulum bob

The acceleration of an object that rotates with variable speed has two components, a
D

centripetal acceleration and a tangential acceleration. We can see this in the above
diagram, fig. 9.1.1, where

Centripetal Acceleration
v2
acent = = ω2 r (9.1.1)
r

Tangential Acceleration
atang = αr (9.1.2)

The net acceleration on the bob can be found by adding the cetripetal and tangential
accelerations
atang + acent = a (9.1.3)
62 GR0177 Exam Solutions

At point (e) At point (c)

a = atang a = acent

Figure 9.1.2: Acceleration vectors of bob at equilibrium and max. aplitude positions

At point (e), v = 0, so

FT
acent = 0 and
atang = a

There is only the tangential component to the bob’s acceleration.


At point (c), α = 0, so
acent = a and
atang = 0
RA
There is only the cetripetal component to the bob’s acceleration.
We see from (C), the acceleration vectors point in the directions we expect.
Answer: (C)

9.2 Coin on a Turntable


The coin will stay in place as long as the certipetal force and the static friction force are
D

equal. We can see that this is dependent on its position on the turntable, see fig. 9.2.1.

Figure 9.2.1: Free Body Diagram of Coin on Turn-Table

The coin is in equilbrium, we see that


mrω2 − µR = 0
µg
∴r= 2 (9.2.1)
ω

David S. Latchman ©2009


Kepler’s Law and Satellite Orbits 63
From here it’s just a matter of plugging what we know, ω = 33.3 revolutions/min. For
the sake of simplicity, we will assume that this is equal to 100/3 revolutions per minute.
Thus
ω = 33.3revolutions/min
100 2π
= ·
3 60
10π
= rad s−1 (9.2.2)
9
The Physics GRE examination is all about estimations. Plugging this into eq. (9.2.1),
we get
µg
r=
ω2

FT
2
9

= 0.3 × 9.8 ×
10π
81
≈3×
900
27
=
100
= 0.27 m (9.2.3)
This is closest to 0.242 m.
RA
Answer: (D)

9.3 Kepler’s Law and Satellite Orbits


Kepler’s Third Law, states, “The square of the orbital period of a planet is directly
proportional to the third power of the semi-major axis of its orbit.” This means
T 2 ∝ r3 (9.3.1)
D

Answer: (D)
See section 1.7.4. If you’re unable to remember Kepler’s Law and its relationship
between the period and orbital distance, some quick calculation will yield some results.
GMm
mRω =
R2
 2
2π GM
mR = 2
T R
2
(2π)
⇒ R3 = T2
GM
∴ R3 = kT2
This is Kepler’s Third Law.

©2009 David S. Latchman


64 GR0177 Exam Solutions
9.4 Non-Elastic Collisions

2m v m 3m vf

Before Collision After Collision

Figure 9.4.1: Inelastic collision between masses 2m and m

ately eliminate choice (A)1 .


Thus
FT
As the bodies fuse, see fig. 9.4.1, the resulting collision will be an inelastic one; kinectic
energy is not conserved but momentum will be conserved. This allows us to immedi-

Momentum Before Collision = Momentum After Collision


RA
2mv = 3mv f
3
⇒ v = vf (9.4.1)
2
The kinectic energy before and after collision are

Initial K.E.
1
E = (2m)v2 = mv2 (9.4.2)
2
D

Final K.E.
 2
1 1 2
E f = (3m)v f = (3m) v
2
2 2 3
2 2
= mv (9.4.3)
3

Subtracting eq. (9.4.3) from eq. (9.4.2) gives us the energy lost in the collision

2
∆E = mv2 − mv2
3
1 2
= mv (9.4.4)
3
1
Not much help but the elimination of just one choice may work to our advantage.

David S. Latchman ©2009


The Equipartition Theorem and the Harmonic Oscillator 65
The fraction of initial kinetic energy lost in the collision is
1 2
mv
∆E 3
=
E mv2
1
= (9.4.5)
3

Answer: (C)

9.5 The Equipartition Theorem and the Harmonic Oscil-


lator

FT
The average total energy of our oscillator is determined by the Equipartition Theorem,
see section 4.22, where !
f
CV = R = 4.16 J mol−1 K−1 (9.5.1)
2
where f is the number of degrees of freedom.
The average total energy is
Q = nCV T
RA
!
f
=n RT
2
!
f
=N kT (9.5.2)
2
We are told this is a three dimensional oscillator so we have f = 6 degrees of freedom
and N = 1. So eq. (9.5.2) becomes
6
 
Q= kT = 3kTJ (9.5.3)
2
D

Answer: (D)

9.6 Work Done in Isothermal and Adiabatic Expansions


This is a simple question if you remember what isothermal and adiabatic processes
look like on a P − V graph. The areas under the curves tell us the work done by these
processes and since the adiabatic curve is less steep than the isothermal one, then the
work done by this process is less2 .
So we get 0 < Wa < Wi .
Answer: (E)
2
Get P-V diagram with isothermal and adiabatic expansions

©2009 David S. Latchman


66 GR0177 Exam Solutions
9.6.1 Calculation
In the event that you’re unable to remember this P − V diagram, you can work out the
equations for the work for isothermal and adiabatic expansions. We are told that
V f = 2Vi (9.6.1)
and for an ideal monatomic gas
γ = 1.66 (9.6.2)
The work done by an expanding gas is
Z Vf
W= P dV (9.6.3)
Vi

Isothermal Work

For an ideal gas

W= FT
PV = nRT
Substituting eq. (9.6.4) into eq. (9.6.3) gives us
Z Vf

Vi
nRT
V
Z Vf
dV

dV
(9.6.4)
RA
= nRT
Vi V
!
Vf
= nRT ln (9.6.5)
Vi
Substituting eqs. (9.6.1) and (9.6.4) into eq. (9.6.5) yields
Wi = PV ln 2 (9.6.6)
This works out to
Wi = 0.69PV (9.6.7)
D

Adiabatic Work

The adiabatic condition is


PV γ = K (9.6.8)
Substituting eq. (9.6.8) into eq. (9.6.3) gives
Z Vf
K
W= γ
dV
Vi V
V
V 1−γ f
=K
1 − γ Vi

h 1−γ 1−γ
i
K V f − Vi
= (9.6.9)
1−γ

David S. Latchman ©2009


Electromagnetic Field Lines 67
Substituting eqs. (9.6.1) and (9.6.8) into eq. (9.6.9) gives

 
PV 21−γ − 1
Wa = (9.6.10)
1−γ

This works out to

Wa = 0.57PV (9.6.11)

This we can see that 0 < Wa < Wi as seen in

Answer: (E)3

9.7 Electromagnetic Field Lines

FT
The two poles are of the same polarity so we expect the filed lines to not cross.4
RA
Answer: (B)

9.8 Image Charges

We can use the “Method of Image Charges” to solve this question. We have a positive
charge near the plate so this will induce an equal and opposite charge in the plate.5
D

But, let’s for the sake of argument say that you didn’t know of this ‘method’ and
needed to figure it out, we know a few things. We know that the plate is grounded.
So if we were to bring a charge near to the plate, an equal but opposite charge will be
induced. In this case, negative charges in the plate are attracted to the nearby charge
and positive ones are repelled. As the positive ones want to “get away”, they succeed
in doing so through the ground, leaving only the negative charges behind. Thus the
plate is left with a net negative charge.

Answer: (D)

3
We can see how relatively easy it is to work this out but it is not something you’ll have time for in
the exam. It’s best to just learn it. The work done by an adiabatic expansion is less than an isothermal
expansion because some of the heat is lost in the temperature change.
4
Get diagram with magnetic field lines
5
Put wikipedia reference here

©2009 David S. Latchman


68 GR0177 Exam Solutions
9.9 Electric Field Symmetry

+q
Gaussian Surface
+q

+q
r
+q
+q

FT
Figure 9.9.1: Five charges arranged symmetrically around circle of radius, r

Gauss’s Law states that “The electric flux through any closed surface is proportional
to the enclosed electric charge”.
I
Qenclosed
E · dA = (9.9.1)
0
RA
S

If we draw a Gaussian Surface at the center of our arrangement, see fig. 9.9.1, we notice
there are no charges enclosed and thus no electric field.6
Answer: (A)

9.10 Networked Capacitors


D

3.0µF 6.0µF 2.0µF

300 V 0V is equivalent to 300 V 0V

Figure 9.10.1: Capacitors in series and its equivalent circuit

The Energy stored in a Capacitor is

1
E = CV 2 (9.10.1)
2
6
This makes sense, there is no electric field inside a conductor because all the charges reside on the
surface.

David S. Latchman ©2009


Thin Lens Equation 69
Our networked capacitors, as shown in fig. 9.10.1, can be reduced to a circuit with only
one capacitor. The equivalence capacitance of two Capacitors in series is
C1 C2
CT =
C1 + C2
(3)(6)
=
9
= 2 µF
Substitute into eq. (9.10.1) we can find the energy stored
1 
E= 2 × 10−6 (300)2
2
= 9 × 10−2 J

FT
Answer: (A)

9.11 Thin Lens Equation


Using the Len’s Maker Equation (Thin Lens). We have
RA
1 1 1
+ 0 = where S = Object Distance
S S f
S0 = Image Distance
f = Focal Length
Solving for the first lens, we have
1 1 1
+ =
40 I1 20
1 1 1
⇒ = −
D

I1 20 40
1
= (9.11.1)
40
The resulting image is 40cm from the first lens which forms a virtual image that is
10cm to the right of the second lens. We get,7
1 1 1
+ =
−10 I2 10
1 1
⇒ = (9.11.2)
I2 5
The image is located 5cm to the right of the second lens.
Answer: (A)
7
Get book/internet references for this equation.

©2009 David S. Latchman


70 GR0177 Exam Solutions
9.12 Mirror Equation

For a concave mirror, we know that if an object is before the focal length, then image
is virtual If object is after focal length, the image is real.
The Mirror Equation is
1 1 1
= + (9.12.1)
f d0 di

We can show that

1 1 1
= + (9.12.2)
F O di
FO

FT
⇒ di = (9.12.3)
O−F

Here we see that O < F, so di is negative. The image is virtual and at point V.
We can also keep in mind that for a concave lens, if the object is between the focal point
and the lens, the image is virtual and enlarged. Think of what happens when you look
at a makeup mirror.8
RA
Answer: (E)

9.13 Resolving Power of a Telescope

The Resolving Power of a Telescope is

λ
sin θ = 1.22 (9.13.1)
D

Solving for D and substuting λ and θ, we get

λ
⇒ D = 1.22
θ
600 × 10−19
= 1.22 ×
3 × 10−5
= 1.22 × 200 × 10−4
= 2.44 × 10−2 m

Answer: (B)

8
Get references for this as well

David S. Latchman ©2009


Radiation detected by a NaI(Tl) crystal 71
9.14 Radiation detected by a NaI(Tl) crystal

dA

First Position Second Position

1m

FT
Figure 9.14.1: Diagram of NaI(Tl) detector postions

Thallium doped Sodium Iodine crystals are used in scintillation detectors, usually
found in hospitals. These crystals have a high light output and are usually coupled to
RA
photomultiplier tubes. No emitted power is lost to the surrounding medium. Thus,
the net power radiated by our source is
Z
P = I dA (9.14.1)

where P is the radiated power, I is the intensity and dA is a differential element of a


closed surface that contains the source.
The emitted power at the first position is
P1 = I1 A1 (9.14.2)
D

and the emitted power at the second position is


P2 = I2 A2 (9.14.3)
As no power is lost to the environment, then eqs. (9.14.2) and (9.14.3) are equal, thus
P1 = P2
I1 A1 = I2 A2
I2 A1
⇒ = (9.14.4)
I1 A2
In the first case, as the detector is right up to the radioactive source, the area, A1 is the
cross sectional area of the NaI(Tl) crystal.
πd2
A1 = (9.14.5)
4

©2009 David S. Latchman


72 GR0177 Exam Solutions
In the second position, the area is the surface area of the sphere of emitted radiation.
A2 = 4πD2 (9.14.6)
where D is the distance of the crystal detector from the radioactive source.
The ratio between I2 and I1 can be found from eq. (9.14.4),
I2 A1
=
I1 A2
πd2 1
=
4 4πD2
d2
=
16D2
2
8 × 10−2
=
16

FT
= 4 × 10−2
Answer: (C)

9.15 Accuracy and Precision


From the graphs, the accuracy is how close the peak is to the reference value while
RA
precision deals with how narrow the peak is. So we look for the graph with the
narrowest peak.
Answer: (A)

9.16 Counting Statistics


From the data given,
√ we have
√ N = 20 counts in T = 10 seconds. Our uncertainty or
counting error is N = 20 . So we can express our uncertainty in the number of
D

counts as √
N = 20 ± 20 (9.16.1)
The rate is the number of counts per unit time. So the uncertainty in the rate is

20 20
R= ± (9.16.2)
10 10
We can see that the error in the rate is δR = δN
T
. Our uncertainty can be expressed
δN
δR T
= N
R T
δN
=
N
1
= √ (9.16.3)
N

David S. Latchman ©2009


Electron configuration 73
The question needs an uncertainty of 1 %. From the above, we see that

1
√ = 0.01 (9.16.4)
N
But N is the number of counts. We want to know how long making these counts will
take us. The count rate, R = NT , thus

1
√ = 0.01 (9.16.5)
2T
Solving for T gives
1
T= = 5000 s (9.16.6)
2(0.01)2

FT
Answer: (D)

9.17 Electron configuration


Standard notation is used to describe the electron configuration of atoms and molecules.
In the case of atoms, the notation is represented by atomic orbital labels with the number
RA
of electrons assigned to each orbital. In the case of phosphorus, which has 15 electrons,
the energy sub-shells are
1s2 , 2s2 , 2p6 , 3s2 , 3p3
Answer: (B)

9.18 Ionization Potential (He atom)


From Bohr’s Theory, the energy needed to completly ionize an atom is
D

Z2
E = −13.6 eV (9.18.1)
n2
The total Energy to ionize the atom is

E = E1 + E2 (9.18.2)

We expect the energy to remove the first electron to be less than the second; there are
more positive charges in the nucleus holding the second electron in place. We can use
the above equation to find out the energy to remove this second electron.

22
E2 = −13.6 2 eV
1
= 54.4eV (9.18.3)

©2009 David S. Latchman


74 GR0177 Exam Solutions
Now we can determine the energy needed to remove the first electron

E1 = E − E2
= 79.0 − 54.4
= 24.6 eV (9.18.4)

Answer: (A)

9.19 Nuclear Fusion


The are several fusion reactions by which stars convert Hydrogen into Helium and
release energy. One of the main reactions is the proton-proton chain reaction and looks
something like this

FT
 
411 H →42 He + 2 01 e +00 γ +0 υ (9.19.1)
Four Hydrogen atoms combine to give one Helium atom, the difference in masses
being released as energy.
Answer: (B)

9.20 Bremsstrahlung X-Rays


RA
Bremsstrahlung is the continuous radiation spectrum of X-Ray radiation that is pro-
duced by a deceleration electron that is deflected off a target metal.
Answer: (E)

9.21 Atomic Spectra


D

The Rydberg Formula describes the spectral wavelengths of chemical elements. For
the Hydrogen atom, the equation is
!
1 1 1
= RH 2 − 2 (9.21.1)
λ n1 n2

where λ is the wavlength of the emitted light, RH is the Rydberg constant for Hydrogen,
n1 and n2 are the electron orbital numbers.
For the Lyman-α emission, electrons jump from n2 = 2to the n1 = 1 orbital. This gives
1 1 1
 
= RH −
λ1 1 2
1
= RH (9.21.2)
2

David S. Latchman ©2009


Planetary Orbits 75
For the Balmer-α emission, electrons jump from n2 = 3 to n1 = 2 orbital. This gives
1 1 1
 
= RH −
λ2 2 3
1
= (9.21.3)
6
Dividing eq. (9.21.3) by eq. (9.21.2), we get
λ1 1
= (9.21.4)
λ2 3

Answer: (C)

FT
9.22 Planetary Orbits
Newton’s Law of Universal Gravitation can be expressed
 2
mv2 Mm 2π
= G 2 = mrω = mr
2
(9.22.1)
r r T
We can use the information above to eliminate choices.
RA
Mass of he Moon We see that in all cases, the mass of the moon, m, cancels out. We
can not find the mass of the moon from the astronomer’s observations.

Mass of the Planet We can determine the mass of the planet, M, from the data.

v2 GM
= 2 (9.22.2)
r r
We will need the distances and the moon’s orbital speed.

Minimum Speed of the Moon The speed of the Moon, v, also does not cancel out in
D

any of our equations. So we can find this out.

Period of Orbit As the period, T, does not cancel out, we can also determine this.
2
2π GM

r = (9.22.3)
T r2

Semi-major axis of orbit The semi-major axis is the longest distance from the center
of an ellipse. The distance, r, in our equations are a measure of the semi-major
axis.

We see that the mass, m, the mass of the moon cancels out. Everything else mentioned
remains. Thus
Answer: (A)

©2009 David S. Latchman


76 GR0177 Exam Solutions
9.23 Acceleration of particle in circular motion

ac

a
θ

at

Figure 9.23.1: Acceleration components of a particle moving in circular motion

FT
Since the particle’s speed increases as it moves in a circle, it is going to have two
accelerations acting on it; a centripetal acceleration and a tangential acceleration, see
fig. 9.23.1. The net acceleration of our particle can be found by adding the centripetal
and tangential components, so
a = ac + at (9.23.1)
where
RA
v2
ac = (9.23.2)
r
at = αr (9.23.3)

The angle between the particle’s acceleration, a and its velocity, v, is


ac
tan θ = (9.23.4)
at
The velocity vector points in the same direction as at , hence the reason why we use the
above calculation.
D

Given that r = 10 meters, v = 10 m s−1 , we calculate ac to be

ac = 10 m s−2 (9.23.5)

Plugging this into eq. (9.23.4), we get


ac
tan θ =
at
10
= =1 (9.23.6)
10
Thus
θ = 45° (9.23.7)

Answer: (C)

David S. Latchman ©2009


Two-Dimensional Trajectories 77
9.24 Two-Dimensional Trajectories
The horizontal velocity of the stone is

Vx = V cos θ (9.24.1)

and the vertical velocity is


V y = V sin θ − gt (9.24.2)
Equation (9.24.1) is constant over time, we choose Graph II and eq. (9.24.2) is a straight
line graph, we choose Graph III.
Answer: (C)

FT
9.25 Moment of inertia of pennies in a circle
RA
2r
r

Figure 9.25.1: Seven pennies in a hexagonal, planar pattern


D

The Moment of inertia of a penny is the same for a disc or cylinder

1
Ipenny = Icm = Mr2 (9.25.1)
2

For the other pennies, we find their Moments of Inertia by using the Parallel Axis
Theorem.
IT = Icm + Md2 (9.25.2)
where d = 2r. This becomes
1
IT = Mr2 + M (2r)2
2
9
= Mr2 (9.25.3)
2

©2009 David S. Latchman


78 GR0177 Exam Solutions
The Moment of Inertia depends on the mass distribution. Whether the six pennies
were stacked on top of each other next or arranged in a hexagonal pattern, the Moment
of Inertia will be the same. The total moment of inertia is found by adding eqs. (9.25.1)
and (9.25.3).

1 9
I = Mr2 + 6 × Mr2
2 2
55
= Mr2 (9.25.4)
2

Answer: (E)

9.26 Falling Rod

L/2
FT
mg
mg
RA
Figure 9.26.1: Falling rod attached to a pivot point

As the rod falls, its Gravitational Potential Energy is converted to Rotational Kinetic
Energy. Ee will need to calculate the Moment of Inertia of the rod about its point of
rotation. For this, we turn to the Parallel Axis Theorem. The Moment of Inertia of the
rod is
1
D

Icm = ML2 (9.26.1)


12
The Parallel Axis Theorem gives us the Moment of Inertia about the pivot point

I = Icm + Md2
 2
1 L
= ML + M
2
12 2
1
= ML2 (9.26.2)
3
The rod is uniform, so its Center of Mass is in the middle of the rod. Its Gravitational
Potential Energy while standing upright is

L
 
PE = Mg (9.26.3)
2

David S. Latchman ©2009


Hermitian Operator 79
The Rotational Kinetic Energy is
1
KE = Iω2 (9.26.4)
2
As energy is conserved, eqs. (9.26.3) and (9.26.4) are equal.
  2
L 1 1 v

Mg = ML2
2 2 3 L
∴ v = 3gL
p
(9.26.5)

Answer: (C)

9.27 Hermitian Operator

FT
The expectation value of an observable Q(x, p) can be expressed
Z
hQi = Ψ∗ Q̂Ψ dx

All measurements have to be real, thus


(9.27.1)
RA
hQi = hQi∗ (9.27.2)

We recall that
hg| f i = h f |gi∗ (9.27.3)
Thus the complex conjugate of an inner product is

hΨ|Q̂ψi = hQ̂Ψ|ψi (9.27.4)

and this must hold for any wave function, Ψ. So operators representing observables
have the property that
D

h f |Q̂ f i = hQ̂ f | f i (9.27.5)


These are called Hermitian.
Answer: (A)

9.28 Orthogonality

Two functions are orthogonal if their inner or dot product is zero . On the other hand
they are orthonormal if their inner product is one. Thus

hψ1 |ψ2 i = 0 (9.28.1)


h1|1i = h2|2i = h3|3i = 1 (9.28.2)

©2009 David S. Latchman


80 GR0177 Exam Solutions
We get,

hψ1 |ψ2 i = (5) (1) h1|1i + (−3) (−5) h2|2i +


+ (2) (x) h3|3i
(9.28.3)

This gives

5 + 15 + 2x = 0
⇒ x = −10 (9.28.4)

Answer: (E)

FT
9.29 Expectation Values
The Expectation Value is defined
Z
hÔi = Ψ∗ ÔΨ dx = hψ|Ôψi (9.29.1)
RA
Where
1 1 1
ψ = √ ψ−1 + √ ψ1 + √ ψ2 (9.29.2)
6 2 3
Thus
1 1 1
hOi = + +
6 2 3
=1 (9.29.3)

Answer: (C)
D

9.30 Radial Wave Functions


We need to be thinking of something that decays exponentially. So

I. e−br This decays exponentially. As r → ∞ then ψ → 0.

II. A sin br This doesn’t go to zero as r → ∞.

III. A/r This does become zero as r → ∞ but there is no realistic value at r = 0. It
blows up.

Answer: (A)

David S. Latchman ©2009


Decay of Positronium Atom 81
9.31 Decay of Positronium Atom
Positronium(Ps) is a quasiatomic structure where an electron, e− , and a positron, e+ ,
are bound together by Coulomb attraction. Positronium has a short lifetime because of
pair-annihilation; the system usually lasts for 10−10 s before decaying into two photons.
As this is a two-body system, we need to find the reduced-mass of the system. This
will allow us to solve this problem as if it was a one body problem. The Reduced Mass
of the System is
m1 m2
µ= (9.31.1)
m1 + m2
Substituting for the masses of the electron and the positron, we get
me mp
µ=
me + mp

FT
me
= (9.31.2)
2
We then apply the Bethe-Salpeter equation,
µ q4e
En = − (9.31.3)
8h2 ε20 n2
In most two body atom systems, the reduced-mass factor is close to unity because the
RA
proton is much heavier than the electron but as the masses of the electron and the
positron are equal, the reduced-mass has an appreciable effect on the energy levels.
Substituting, we get,

1 me q4e 1
En = −
2 8h2 ε20 n2
−6.8
= (9.31.4)
2
= −3.4 eV (9.31.5)
D

Answer: (A)

9.32 Relativistic Energy and Momentum


The rest energy of our particle is
Erest = mc2 (9.32.1)
The particle’s total energy is given by

E2 = c2 p2 + m2 c4 (9.32.2)

We are told that


E = 2Erest (9.32.3)

©2009 David S. Latchman


82 GR0177 Exam Solutions
Substuting eq. (9.32.3) in eq. (9.32.2) gives
E = 2Erest
∴ 4m c = p2 c2 + m2 c4
2 4

⇒ p = 3 mc (9.32.4)

Answer: (D)

9.33 Speed of a Charged pion


The speed that is measured is the same in either frame, so we can say
L L0

FT
v= = 0 (9.33.1)
∆t ∆t
We know that from the pi-meson’s point of view the distance is length contracted.
r
v2
L0 = L 1 − 2 (9.33.2)
c
We can alternatively look at things from the laboratory’s point of view, in this case
we will be using the Relativistic Time Dilation Formula. Substuting eq. (9.33.2) into
RA
eq. (9.33.1), gives us r
L v2
v= 1− 2 (9.33.3)
∆t c
With some manipulation, we simplify eq. (9.33.3) to give
L2
v2 = .c2
+
L2 c2 (∆t0 )2
900
=
900 + 9
100
= (9.33.4)
D

101
We can surmise that the result of eq. (9.33.4) will be closer to (D) than (C).
Answer: (D)

9.34 Simultaneity
The Space-Time Interval of an event is
∆S2 = ∆x2 + ∆y2 + ∆z2 − c2 ∆t2 (9.34.1)
The Space-Time interval is a Lorentz-invariant quantity which means that it has the
same value in all Lorentz frames. Depending on the two events, the interval can be
positive, negative or zero. So

David S. Latchman ©2009


Black-Body Radiation 83
Time-Like If ∆S < 0, the two events occur in the same space but at different times.

Space-Like If ∆S > 0, the two events occur at the same time (simultaneously) but are
seperated spatially.

Light-like If ∆ = 0, the two events are connected by a signal moving at light speed.

So we are looking for a situation where ∆S is positive. Thus

∆S = ∆x2 − c2 ∆t2 > 0


∆x
⇒ >c (9.34.2)
∆t
Answer: (C)

9.35 Black-Body Radiation

FT
The energy radiated by a black body is given by the Stefan–Boltzmann’s Law

Let
u = σT4 (9.35.1)
RA
u1 = σT14 (9.35.2)

The temperature of the object increases by a factor of 3, thus

T2 = 3T1
u2 = σ (3T1 )4 (9.35.3)

and we get

u2 = 81σT14
D

= 81u1 (9.35.4)

It increases by a factor of 81.


Answer: (E)

9.36 Quasi-static Adiabatic Expansion of an Ideal Gas


This is more of a defintion question that we can best answer through the process of
elimination.

(A) This is TRUE. The expansion is quasi-static, which means that it happens very
slowly and hence at equilibrium. Hence no heat is exchanged.

©2009 David S. Latchman


84 GR0177 Exam Solutions
(B) Again this is TRUE. The entropy is defined

dQ
dS = (9.36.1)
T

As dQ = 0, then there is no change in entropy.

(C) The First Law of Thermodynamics says

dU = −dW + dQ (9.36.2)

where dQ = 0, we see that


dU = −dW (9.36.3)
The work done by the gas is

FT
Z
dW = pdV (9.36.4)

Thus Z
dU = − PdV (9.36.5)

(D) We see from eq. (9.36.4) that this is TRUE.

(E) The temperature of the gas is not constant. For an adiabatic process
RA
PV γ = constant (9.36.6)

Given PV = nRT, substituting this into the above equation gives

TV γ−1 = constant (9.36.7)

So
γ−1 γ−1
Ti Vi = Tf Vf (9.36.8)
The temperature of the gas is not constant. So this is NOT TRUE.
D

Answer: (E)

9.37 Thermodynamic Cycles


Thermodynamic Work is defined
Z Vf
W=− PdV (9.37.1)
Vi

The Total Work is calculated along each path. Thus,

W = WC→A + WA→B + WB→C (9.37.2)

David S. Latchman ©2009


RLC Resonant Circuits 85
Work along path C → A is an isochoric process, dV = 0.
WC→A = 0 (9.37.3)
Work along path A → B is an isobaric process
WA→B = P · dV
= P (VB − 2) (9.37.4)
Work along path B → C is an isothermal process
Z VC
nRT
WB→C = PdV where P=
VB V
Z VA
dV
= nRT
VB V

FT
VA
 
= nRT ln (9.37.5)
VB
The path along BC is an isotherm and this allows us to find the volume at point B.
PB VB = PC VC = nRT
200VB = 500 · 2
∴ VB = 5 (9.37.6)
RA
Plugging in what we know, we add eq. (9.37.4), eq. (9.37.5) and eq. (9.37.3) to get the
total work.
W = WC→A + WA→B + WB→C
VA
 
= 0 + P (VB − VA ) + PC VC ln
V
  B
2
= 200 (5 − 2) + (500) (2) ln
5
2
 
= 600 + 1000 ln (9.37.7)
5
D

Now ln(2/5) > −1, so we expect


2
 
W = 600 + 1000 ln > −400 kJ (9.37.8)
5

Answer: (D)

9.38 RLC Resonant Circuits


The current will be maximized when the inductive and capacitive reactances are equal
in magnitude but cancel each other out due to being 180°out of phase. The Inductive
Impedance is
XL = ωL (9.38.1)

©2009 David S. Latchman


86 GR0177 Exam Solutions
And the Capacitive Impedance is
1
XC = (9.38.2)
ωC
We let, eq. (9.38.2) = eq. (9.38.1)

1
ωL =
ωC
1
⇒C= 2
ωL
1
=
25 × 10−3
= 40µF (9.38.3)

Answer: (D)

9.39 High Pass Filters


We recall that the Inductive Impedance is
FT
XL = ωL (9.39.1)
RA
and the Capacitive Impedance to be

1
XC = (9.39.2)
ωC
If we look at eq. (9.39.1), we see there is a linear relationship between L and XL ; an
increase in L results in an increase in XL .
We also see from eq. (9.39.2) that there is an inverse relationship between C and XC ; an
increase in C decreases XC .
D

Recall the Voltage Divider Equation

X2
VOut = VIn (9.39.3)
X1 + X2

We will use this to help us solve the question.

Circuit 1 As ω increases, the impedance of the inductor increases; X1 becomes large.


Think of this as a very large resistor where the inductor is. We see from eq. (9.39.3)

X2
VOut = VIn
∞ + X2
= 0VIn (9.39.4)

. This is a Low-Pass Filter.

David S. Latchman ©2009


RL Circuits 87
Circuit II As ω increases, XL becomes large. We will say that X2 >> X1 . Again
eq. (9.39.3) shows that

X2
VOut = VIn
X1 + X2
1
≈ VIn (9.39.5)
1
This is one of the High-Pass Filters.

Circuit III As ω increases, XC decreases. At high ω, XC = X1 ≈ 0. Again eq. (9.39.3)


shows that
X2
VOut = VIn
0 + X2

FT
= VIn (9.39.6)

This is the other High-Pass Filter.

Circuit IV Using what we know, eq. (9.39.3) tells us that

0
VOut = VIn
X1 + 0
=0 (9.39.7)
RA
This is a Low-Pass Filter.

Thus Circuits II & III are our High-Pass Filters.


Answer: (D)

9.40 RL Circuits
D

As an EMF is introduced in the circuit, there is going to be a slowly rising (or falling)
current. If the inductor was not present, the current would rapidly rise to a steady
state current of ER . The inductor produces a self-induced EMF, EL , in the circuit; from
Lens’s Law. This EMF is
di
EL = −L (9.40.1)
dt
Applying Kirchoff’s Voltage law gives

di
E = iR + L (9.40.2)
dt
This differential equation can be solved such that

E  Rt

i= 1 − e− L (9.40.3)
R

©2009 David S. Latchman


88 GR0177 Exam Solutions
Let τL = RL , we can rewrite eq. (9.40.3), to say

E  − t

i= 1 − e τL (9.40.4)
R
1
The time constant is the time to fall to e
of its original value.

10mH
τL =
2Ω
= 2 milli-seconds (9.40.5)

As we expect the EMF to decay across the inductor, we choose


Answer: (D)9

9.41 Maxwell’s Equations

FT
Maxwell’s Equations relate electric and magnetic fields to the motion of electric charges.
These equations allow for electric charges and not for magnetic charges. One can write
symmetric equations that allow for the possibility of “magnetic charges” that are similar
to electric charges. With the inclusion of these so called “magnetic charges”, ρm , we
RA
must also include a magnetic current, jm . These new Maxwell equations become

Gauss’ Law This equation relates the distribution of electric charge to the resulting
electric field.
∇ · E = 4πρe (9.41.1)

Gauss’ Law for Magnetism Here we assume that there are no magnetic charges, so
the equation that we know and have studied is

∇·B=0 (9.41.2)
D

But if we assume for magnetic charges, the equation will become

∇ · B = 4πρm (9.41.3)

Here we have used a symmetric argument from Gauss’ Law to get this equation.

Ampère’s Law This equation relates the magnetic field to a current. With Maxwell’s
displacement current, je , we have

∂E
∇×B= + 4πje (9.41.4)
∂t
9
We note that choices D) and E) both decay exponentially and that milli-second decay times are
standard with the usual components you find in a lab. A 200 sec decay time is unusual given the
“normal” electronic components in the question.

David S. Latchman ©2009


Faraday’s Law of Induction 89
Faraday’s Law of Induction This equation relates a changing Magnetic field to an
Electric Field. The equation is

∂B
∇×E= (9.41.5)
∂t
Again we will use a symmetric argument to “derive” the magnetic monopole
case. As in Ampère’s Law where there exists an electric displacement current,
we postulate a “magnetic displacement current”. This becomes

∂B
∇×E= + 4πjm (9.41.6)
∂t

We see the changes are in equations II and III.

FT
Answer: (E)

9.42 Faraday’s Law of Induction


Faraday’s Law states that the induced EMF is equal to the rate of change of magnetic
flux.
dΦB
E =−
RA
(9.42.1)
dt
The minus sign is from Lenz’s Law which indicates that the induced EMF and the
changing magnetic flux have opposite signs. Thinking of it as a system that induces
an opposing force to resist the change of this changing flux is one of the best analogies.

Loop A In this case, the flux increases as the current carrying loop approaches. So
to ‘compensate’ for this increase, Loop A, induces a current in the opposite
direction to prevent this increase. Thus the induced current wil be in the clock-
wise direction.
D

Loop B As the current carying loop moves away from Loop B, the magnetic flux
will decrease. Loop B wants to prevent this decrease by inducing an increasing
current. The induced current will be in the clock-wise direction.

Answer: (C)

9.43 Quantum Mechanics: Commutators


We recall our commutator relations

[B, AC] = A [B, C] + [B, A] C (9.43.1)


[A, B] = − [B, A] (9.43.2)

©2009 David S. Latchman


90 GR0177 Exam Solutions
Thus
h i h i
Lx L y , Lz = − Lz , Lx L y
 h i 
= − −Lx Lz , L y + [Lz , Lx ] L y
   
= − −Lx (i~Lx ) + i~L y L y
 
= i~ L2x + L2y (9.43.3)

Answer: (D)

9.44 Energies

FT
We are given that
n2 π2 ~2
En = (9.44.1)
2mL2
where n = 1, 2, 3, · · · So the possible energy values are E2 = 4E1 , E3 = 9E1 , E4 = 16E1 , · · · .
Possible answers are of the form
En = n2 E1 (9.44.2)
RA
textbfD) follows where n = 3. All the rest don’t.10
Answer: (D)

9.45 1-D Harmonic Oscillator

We are given that


1
 
H|ni = ~ω n + (9.45.1)
D

2
and that
1 2 3
|ψi = √ |1i − √ |2i + √ |3i (9.45.2)
14 14 14
For the Energy eigenstates, we calculate

3
H|1i = ~ω|1i (9.45.3)
2
5
H|2i = ~ω|2i (9.45.4)
2
7
H|3i = ~ω|3i (9.45.5)
2
10
This question seems to be designed to trip you up and make you focus on irrelevant details.

David S. Latchman ©2009


de Broglie Wavelength 91
The Expectation Value is

1 3 4 5 9 7
hψ|H|ψi = ~ω + ~ω + ~ω
14 2 14 2 14 2
3 20 63
= ~ω + ~ω + ~ω
28 28 28
43
= ~ω (9.45.6)
14
Answer: (B)

9.46 de Broglie Wavelength

FT
The Energy of a particle can be related to its momentum by

p2
E= (9.46.1)
2m
The de Broglie Relationship is
h
λ= (9.46.2)
p
RA
Substituting eq. (9.46.1) into eq. (9.46.2), yields

h
λ= √ (9.46.3)
2mE
The particle enters a region of potential, V. So

E0 = E − V (9.46.4)

This changes the de Broglie Wavelength of the particle such that


D

h
λ0 = √ (9.46.5)
2mE0
Dividing eq. (9.46.5) by eq. (9.46.3), yields

λ0 h 2mE
= p
λ 2m (E − V) h

λ 2mE
⇒λ = p
0

2m (E − V)
 1
V −2

=λ 1− (9.46.6)
E
Answer: (E)

©2009 David S. Latchman


92 GR0177 Exam Solutions
9.47 Entropy
Our container is sealed and thermall insulated. This means that the temperature
throughout the process remains the same. You may recall that the work done by an
isothermal process is
" #
Vf
W = nRT ln (9.47.1)
Vi
Where V f = 2Vi . As the temperature remains the same, there is no change in internal
energy. The First Law of Thermodynamics says
dU = −dW + dQ (9.47.2)
where dU = 0, so
dQ = dW (9.47.3)

FT
The Entropy of a system is defined as
dQ
dS = (9.47.4)
T
eq. (9.47.4) becomes
nRT ln (2)
dS =
T
RA
= nR ln 2 (9.47.5)

Answer: (B)

9.48 RMS Speed


The vrms of a gas is r
3RT
vrms = (9.48.1)
M
D

There is an inverse relationship between the rms speed,vrms , and the molar mass, M.
The Molar Masses of Oxygen and Nitrogen are 64u and 56u respectively.
r
1
vrms ∝
M
s
vrms (N2 ) MO2
⇒ =
vrms (O2 ) MN2
r
64
=
56
r
8
= (9.48.2)
7
Answer: C)

David S. Latchman ©2009


Partition Function 93
9.49 Partition Function

The Partition Function is defined


X
Z= g j · e−βE j (9.49.1)
j

where
1
β=
kB T
g j = degeneracy for each state

So

FT
 2
− −
Z = 2e kB T + 2e kB T
  
−k T − k2T
=2 e B +e B (9.49.2)

Answer: (E)
RA
9.50 Resonance of an Open Cylinder

We don’t need to recall the resonance formula for an Open Cylinder to solve this
problem. We do need to realize that the wavelength of the soundwave will remain
the same as we are assuming that the dimensions of the cylinder will not change. We
know
v = fλ (9.50.1)
At 20°C, we have
D

v1 = f1 λ (9.50.2)
The speed of sound is 3% lower, so

v2 = 0.97v1 (9.50.3)

The resonant frequency of the pipe on a cold day becomes

v2 = f2 λ
0.97v1 = f2 λ
f2 = 0.97 f1
= 427Hz (9.50.4)

Answer: (B)

©2009 David S. Latchman


94 GR0177 Exam Solutions
9.51 Polarizers
The Law of Malus gives us the intensity of a beam of light after it passes through a
polarizer. This is given by
I = I0 cos2 θi (9.51.1)
A beam of light is a mixture of polarizations at all possible angles. As it passes through
a polarizer, half of these vectors will be blocked. So the intensity after passing through
the first polarizer is
I1 1
= (9.51.2)
I0 2
Each polarizer reduces the intensity of the light beam by a factor of 12 . With n polarizers,
we can say
 n
In 1
=

FT
(9.51.3)
I0 2
Where n = 3, we substitute into eq. (9.51.3) and get
 3
I 1
=
I0 2
1
= (9.51.4)
8
RA
Answer: (B)

9.52 Crystallography
We are told that the volume of the cube is

V = a3 (9.52.1)

For a cube, each corner has 1/8 of an atom. In the BCC case, we also have an atom in
D

the center. So there are a total of two atoms in our BCC crystal’s primitive unit cell.
The volume of this primitive unit cell is V/2 = a3 /2.
Answer: (C)

9.53 Resistance of a Semiconductor


To best answer this it helps to know some things about semiconductors. Semicon-
ductors are closer to insulators than conductors, the only difference being their energy
levels. Typically, an insulator requires a lot of energy to break an electron free from an
atom (typically about 10eV), while a semiconductor requires about 1eV.
When a semiconductor is ‘cold’, all its electrons are tightly held by their atoms. When
the substance is heated, the energy liberates some electrons and he substance has some

David S. Latchman ©2009


Impulse 95
free electrons; it conducts. The more energy the more electrons freed. So we are looking
for a relationship where the conductivity increases with temperature.
This contrasts with a conductor whose resistivity decreases with increasing temperature.
This property has to do with the conductor’s free electrons, as temperature increases,
the atoms vibrate more and increases the number of collisions with any moving free
electrons. As a result it’s more difficult for the electrons to move through the conductor
and the resistivity increases.
Answer: (A)

9.54 Impulse

FT
The Impulse is defines as Z
J= F dt (9.54.1)

On a F vs. t graph, the Impulse will be the area under the curve.
The area under the graph is thus

2×2
J= = 2 kg m s−1 (9.54.2)
RA
2

Answer: (C)

9.55 Fission Collision

Once masses split up or fuse energy is not conserved but we know that momentum is
always conserved. Horizontal Momentum
D

mv = 2mv0 cos θ
v
⇒ v0 = (9.55.1)
2 cos θ
Vertical Momentum
0 = mv0 sin θ − mv0 sin θ (9.55.2)
The value of θ can be
0° 6 θ (9.55.3)
Plugging eq. (9.55.3) into eq. (9.55.1), and we see that
v
v0 > (9.55.4)
2
Answer: (E)

©2009 David S. Latchman


96 GR0177 Exam Solutions
9.56 Archimedes’ Principal and Buoyancy

FT
Mg

mg
RA
Figure 9.56.1: Diagram of Helium filled balloon attached to a mass

Archimedes’ Principle states that when an object is fully or partiall immersed in a fluid,
the upthrust acting on it is equal to the weight of fluid displaced. If we neglected the
weight of the balloon, we see from fig. 9.56.1, for the helium balloon to just float our
mass
U − Mg − mg = 0 (9.56.1)
where U is the upthrust, M is the mass of helium and m is the mass to be suspended.
Given the density of helium, ρHe = 0.18 kg m−3 and the density of air, ρair = 1.29 kg m−3 ,
D

we have
U = ρair Vg (9.56.2)
and
M = ρHe g (9.56.3)
where V is the volume of Helium used and air displaced. Substituting eqs. (9.56.2)
and (9.56.3) into eq. (9.56.1) and simplifying, we get
m
V= (9.56.4)
ρair − ρHe
which works out to be
300
V=
1.29 − 0.18
= 270 m3 (9.56.5)

David S. Latchman ©2009


Fluid Dynamics 97
Answer: (D)

9.57 Fluid Dynamics

The force on the wall is found from Newton’s Second Law


dp
F= (9.57.1)
dt
The momentum is defined as
p = mv (9.57.2)
So

FT
dp
F=
dt
=0
z}|{
= m · dv +v · dm
= v · dm (9.57.3)

We need to calculate dm, the density of fluid is


RA
M dm
ρ= = (9.57.4)
V dV
Substituting this into eq. (9.57.3), we get

dV
F = vρ
dt
dx
= vρA
dt
D

= v2 ρA (9.57.5)

Answer: (A)

9.58 Charged Particle in an EM-field

The Forces on a negatively charged particle in Electric and Magnetoic Fields are de-
scribed by the Lorentz law.
F = q (E + (v × B)) (9.58.1)
In the first case, the electron is undeflected, so we can write

F1 = e [E + v1 × B] = 0 (9.58.2)

©2009 David S. Latchman


98 GR0177 Exam Solutions
Vectorially our directions are
E = Eî (9.58.3)
v = vk̂ (9.58.4)
B = Bĵ (9.58.5)
eq. (9.58.2) becomes
h  i
F1 = e Eî + v1 k̂ × Bĵ
h  i
= e Eî + v1 B k̂ × ĵ
h i
= e Eî − v1 Bî
=0 (9.58.6)
eq. (9.58.6) is balanced so

FT
E − v1 B = 0 (9.58.7)
We are told that the accelerating potential is doubled, so the speed at which the electron
enters is
1 2
mv = eV
2 1 r
2eV
⇒ v1 =
m
RA

∴ v2 = v1 2 (9.58.8)
Since v2 > v1 , we can see that
h √ i
F2 = e Eî − v1 B 2 1̂ < 0î (9.58.9)
The electron will move in the negative x-direction.
Answer: (B)

9.59 LC Circuits and Mechanical Oscillators


D

We are given
1
LQ̈ + Q=0 (9.59.1)
C
For a mechanical oscillator,
mẍ + kx = 0 (9.59.2)
Comparing both equations we see that
L=m (9.59.3)
1
=k (9.59.4)
C
and Q = x (9.59.5)
Answer: (B)

David S. Latchman ©2009


Gauss’ Law 99
9.60 Gauss’ Law

Gauss’ Law states that the electric flux through any Gaussian surface is proportional
to the charge it encloses. I
QEnclosed
E · dA = (9.60.1)
0
The charge density, σ, is
Q
σ= (9.60.2)
A
We need to find the area that the Gaussian Surface encloses on the carged sheet. The
Gaussian Surface encompasses a circle of radius (R2 − x2 ). So the charge enclosed is

QEnclosed = σA

FT
 
= σπ R2 − x2 (9.60.3)

The Electric Flux is


QEnclosed
Φ=
0
σπ R2 − x2

= (9.60.4)
0
RA
Answer: (C)

9.61 Electromagnetic Boundary Conditions

The boundary conditions for Electrodynamics can be expressed

1 E⊥1 − 2 E⊥2 = σ Ek1 − Ek2 =0


D

1 k 1
B⊥1 − B⊥2 = 0 B1 − Bk2 =0
µ1 µ2

We are given
E = E0 cos(kx − ωt) (9.61.1)
We are told that we have a perfect conductor

9.62 Cyclotron Frequency

The cetripetal force is equal to the transverse magnetic field. So

BQv = mrω2 (9.62.1)

©2009 David S. Latchman


100 GR0177 Exam Solutions
Solving for m, gives
BQ
m= (9.62.2)
2π f
Plugging what we know, we get

π 2 × 1.6 × 10−19
m= × kg
4 2 × π × 1600
= 2.5 × 10−23 kg (9.62.3)

Answer: (A)

9.63 Wein’s Law

FT
Wein’s Law tells us there is an inverse relationship between the peak wavelength of a
blackbody and its temperature. It says

λMax T = const. (9.63.1)

From the graph, we see that the peak wavelength is approximately 2µm. Plugging this
RA
into eq. (9.63.1), we get

2.9 × 10−3
T=
2.0 × 10−6
= 1.45 × 103 K (9.63.2)

Answer: (D)

9.64 Electromagnetic Spectra


D

This question tests your knowledge of Electromagnetic Radiation and its properties.

A Infra-red, Ultraviolet and Visible Light emissions occur at the electron level. You
would typically expect higher EM radiation levels to occur at the nuclear level.
NOT CORRECT

B The wavelengths in the absorbtion spectra are the same for emission. They are in a
sense, the negative image of each other. Correct

C This is true. We do analyse the spectral output of stars to determine its composition.
Correct

D Again this is also true. Once it interacts with photons we can detect it. Correct

David S. Latchman ©2009


Molar Heat Capacity 101
E Molecular Spectral lines are so close to each other they often appear to be bands of
‘color’; their interaction is much richer than that of elements which often appear
as spectral lines. Correct

So by the process of knowledgeable elimination, we have


Answer: (A)

9.65 Molar Heat Capacity


We are given that Einstein’s Formula for Molar Heat Capacity is
!2 hυ
hυ e kT
C = 3kNA (9.65.1)

FT
kT e kT

2
−1

We recall that
ex ≈ 1 + x (9.65.2)
So we can simplify
e kT

1 + kT

 hυ 2 =  2 (9.65.3)

e kT −1 kT
RA
Plugging in eq. (9.65.3) into eq. (9.65.1), we have
" #

C = 3kNA 1 + (9.65.4)
kT

As T → ∞, kT
→ 0,
C = 3kNA (9.65.5)

9.66 Radioactive Decay


D

The total decay rate is equal to the sum of all the probable decay rates. If you didn’t
know this, some quick calculation would show this. So for an exponential decay
dN
= −λN (9.66.1)
dt
Solving this, we have
N = N0 e−λT (9.66.2)
Let’s say that there are two decay modes or channels along which our particle can
decay, we have
dN
= −λ1 N − λ2 N
dt
= −N (λ1 + λ2 ) (9.66.3)

©2009 David S. Latchman


102 GR0177 Exam Solutions
Solving gives us,
N = N0 e−(λ1 +λ2 )T (9.66.4)
From the above two equations, we can see that we add the decay constants,
λC = λ1 + λ2 + · · · (9.66.5)
We also know that the mean time, τ, is
1
τ= (9.66.6)
λ
So
1 1 1
= + (9.66.7)
τC τ1 τ2
This gives us
1 1 1
= +
τ 24 36

FT
τ1 τ2
τ=
τ1 + τ2
24 · 36
=
24 + 36
= 14.4 (9.66.8)
Answer: (D)
RA
9.67 Nuclear Binding Energy
Nuclei are made up of protons and neutrons but the sum of the individual masses is
less than the actual mass of the nucleus. The Energy of this ‘missing’ mass is what
holds the nucleus together and is known as the Binding Energy. As a heavy nucleus
splits or undergoes fission, some of this energy is released.
Ui − U f = K = 200 MeV (9.67.1)
So
D

U f = Ui − K
= 238(7.8) − 200 (9.67.2)
Equation (9.67.2) refers to the total energy holding the nucleus together. To find
the Binding Energy per nucleon we divide U f by 238. To make this simpler and to
save precious time, let’s say there were 240 nucleons and their binding energy was 8
Mev/nucleon11 Thus
(240)(8) − 200
(9.67.3)
240
We can see the binding energy for a nucleus, A = 120, is less than 8MeV/nucleon.
Answer: (D)
11
This actually works out to be about 6.96MeV/nucleon. Binding Energy peaks around Iron which
has a binding energy of 8.8 MeV/nucleon and an atomic mass, A = 55. If you knew this you won’t have
had to work anything out.

David S. Latchman ©2009


Radioactive Decay 103
9.68 Radioactive Decay

We are told that Beryllium decays to Lithium. By looking at this process we expect it
to be some sort of β-decay process. Electron Capture is a type of a β-decay. The decay
looks like this
4 Be +−1 e −→3 Li + υ
7 0 7
(9.68.1)

where υ is a neutrino.
Answer: (E)

9.69 Thin Film Interference

FT
Since the refractive index of glass is higher than that of oil, the maxima can be found
by
2nLmin = mλ

where n is the refractive index of the oil film and m to its order. Thus

2(1.2)Lmin = 1(480 × 10−9 )


(9.69.1)
RA
∴ Lmin = 200 × 10−9 m (9.69.2)

Answer: (B)

9.70 Double Slit Experiment

Young’s Double Slit Equation states for constructive interference


D

d sin θ = mλ (9.70.1)

If we take into account the distance, D, of the screen and finge seperation, ∆y, we get

mλD
∆y = (9.70.2)
d

In terms of frequency, υ, there is an inverse relationship with the fringe seperation.

mcD
∆y = (9.70.3)

So doubling the frequency, υ, will half the finge seperation.


Answer: (B)

©2009 David S. Latchman


104 GR0177 Exam Solutions
9.71 Atomic Spectra and Doppler Red Shift
The Relativistic Dopler Shift (Red) is
s
1−β
ῡ = υ0 (9.71.1)
1+β
q
1−β
where β = u
c
and let r = 1+β

121.5r = 607.5
r≈5
12
∴v≈ c (9.71.2)

FT
13
As the wavelength is longer, it is red-shifted and thus moving away from the observer.
We also expect the speed to be close to c.
Answer: (D)

9.72 Springs, Forces and Falling Masses


RA
Before the string is cut, the system is in equilibrium. Let us call the tension on the top
string, T1 and the tension on the spring, T2 . Let’s also call the top and bottom masses,
M1 and M2 respectively. Thus

T1 − M1 g − M2 g = 0 (9.72.1)
T2 = M2 g (9.72.2)

After the string is cut, T1 is now zero. The spring, pulls on the top mass with the froce
D

due to its extension. Thus

M1 a = −M1 g − T2
= −M1 g − M2 g
⇒ a = −2g (9.72.3)

Answer: (E)

9.73 Blocks and Friction


The Force used to push the blocks is

F = (MA + MB ) a (9.73.1)

David S. Latchman ©2009


Lagrangians 105
The Reaction on the block, B, is
R = MB a (9.73.2)
Since block B doesn’t move, we can say

MB g − µR = 0 (9.73.3)

Substituting eq. (9.73.1) and eq. (9.73.2) into the equation gives us

F
 
MB g − µMB =0
MA + MB

g (MA + MB )
⇒F= (9.73.4)
µ

FT
We get F = 40g
Answer: (D)

9.74 Lagrangians
RA
The Langrangian for the system is

L = aq̇2 + bq4 (9.74.1)

The equation of motion is


∂L d ∂L
!
= (9.74.2)
∂q dt ∂q̇
Solving, we get
∂L
= 4bq3
D

(9.74.3)
∂q

d ∂L
!
= 2aq̈ (9.74.4)
dt ∂q̇

eq. (9.74.3) is equal to eq. (9.74.4)

2aq̈ = 4bq3
2b
q̈ = q3 (9.74.5)
a

Answer: (D)

©2009 David S. Latchman


106 GR0177 Exam Solutions
9.75 Matrix Transformations & Rotations

We notice that in the transformation matrix, that a33 = 1. The coordinate in the z-axis
remains unchanged and thus we expect a rotation about this point. The Rotation
Matrix about the z-axis is of the form

 cos θ sin θ 0 
 
A =  − sin θ cos θ 0  (9.75.1)
 
0 0 1
 

From the above, we see that

1
cos θ = (9.75.2)
2√

FT
3
sin θ = (9.75.3)
2
Solving
θ = 60◦ (9.75.4)
As positive rotations are in the counter-clockwise direction
Answer: (E)
RA
9.76 Fermi Gases

Electrons are fermions and follow Fermi-Dirac Statistics. This means that they follow
the Pauli Exclusion Principle; every fermion must have a unique quantum state. This
means that the total energy of the Fermi gas at zero temperature will be larger than
the product of the number of particles and the single-particle ground state energy; the
fermions will occupy all states from ground state up until all the quantum states are
occupied.
D

Answer: ((C)

9.77 Maxwell-Boltzmann Distributions

Recall the Partition Function X 


Z= g j · e− kT (9.77.1)
j

The degeneracies, g j , are the same for both states. So the ratio between bot states
becomes
b +0.1
Za = e− kT
0.1
(9.77.2)
Zb = e− kT

David S. Latchman ©2009


Conservation of Lepton Number and Muon Decay 107
Simplifying, this becomes
Za 0.1
= e− kT
Zb
= e−4 (9.77.3)

Answer: (E)

9.78 Conservation of Lepton Number and µ− Decay


Muons are elementary particles, similar to electrons that decay via the weak interaction

µ− = e− + υµ + υe (9.78.1)

FT
We can analyze each choice in turn and eliminate

Charge The muon is best described as a heavy electron. The neutrino on the other
hand has no charge. So the charges in the above reaction are

− 1 = −1 + 0 (9.78.2)

They balance, so charge is conserved.


RA
Mass This one is a bit trickier. We know the muon mass is about 200 times the
electron. Neutrinos on the other hand have a small but nonzero mass. Is it
enough to complete this reaction? Maybe
Energy and Momentum It may be possible to kinematically make this work.
Baryon Number Baryons are a list of composite particles; they are made up of quarks.
Muons, neutrinos, and the other particles mentioned in this question are elemen-
tary particles. Thus, the Baryon number for all are, B = 0. So

0=0+0
D

(9.78.3)

So there is conservation of Baryon number.


Incidentally, the Baryon number can be found by knowing the component quarks
and antiquarks.
nq − nq̄
B= (9.78.4)
3
where nq is the number of constituent quarks and nq̄ the number of constituent
antiquarks.
Lepton Number There are several ways that lepton number must be conserved in
a reaction. We can add/subtract the number of leptons and antileptons at the
beginning and end of the reaction and see if it is conserved.

Answer: ((E)

©2009 David S. Latchman


108 GR0177 Exam Solutions
9.79 Rest Mass of a Particle
The Relativistic Energy is the sum of its Rest Mass, m0 , and its momentum, p. Thus
q
E = p2 c2 + m20 c4 (9.79.1)

We are given that

E = 10 GeV
p = 8 GeV/c

Substituting E and p into eq. (9.79.1), we have

102 = 82 + m20 c2

FT
100 − 64 = m20 c2
⇒ m0 = 6 GeV/c2 (9.79.2)

Answer: (D)

9.80 Relativistic Addition of Velocities


RA
The Relativistic Addition of Velocities is
u0s + v
ux = (9.80.1)
u0 v
1 + s2
c
Substituting
c
v= (9.80.2)
2
D

c 3c
u0x = = (9.80.3)
n 4
We get
10c
ux = (9.80.4)
11
Answer: (D)

9.81 Angular Momentum


The orbital angular momentum is

L2 = ` (` + 1) ~2 (9.81.1)

David S. Latchman ©2009


Addition of Angular Momentum 109
and the z-component of the angular momentum in terms of the magnetic quantum
number is
Lz = m` ~ (9.81.2)
We are told that

L2 = 6~2 (9.81.3)
Lz = −~ (9.81.4)

Solving for the above equations gives us

` (` + 1) = 6 (9.81.5)
m = −1 (9.81.6)

Solving for `, gives

FT
` = −3; 2 (9.81.7)

 negative numbers for `, so we are left with ` = 2 and m = −1.


It’s not possible to have
This gives us Y2 θ, φ .
−1

Answer: (B)
RA
9.82 Addition of Angular Momentum

Addition of Angular Momentum


NOT FINISHED pp.185

9.83 Spin Basises


D

Spin Basises NOT FINISHED

9.84 Selection Rules

The selection rules for an electric-dipole transition are

∆` = ±1 Orbital angular momentum


∆m` = 0, ±1 Magnetic quantum number
∆ms = 0 Secondary spin quantum number
∆ j = 0, ±1 Total angular momentum

We can examine the transitions to see which are valid

©2009 David S. Latchman


110 GR0177 Exam Solutions
Transition A This transition goes from

n=2 to n=1

and
`=0 to `=0
We see that this transition is forbidden12 .

Transition B This transition goes from

`=1 to `=0

and
3 1
j= to j=

FT
2 2
This leaves us with the transitions

∆` = −1 and ∆j = −1

These are allowed transitions.

Transition C The transition goes from


RA
1 1
j= to j=
2 2
This leaves us with the transition

∆j = 0

Which is a valid transition13 .

From the above, we see that transitions B & C are the only valid ones.
D

Answer: (D)

9.85 Resistivity

Ohm’s Law gives the Resistance to be

ρL
R= (9.85.1)
A
12
Not forbidden really, just highly unlikely.
13
∆j = 0 is a valid transition as long as you don’t have the j = 0 → j = 0 transition. The vector angular
momentum must change by one unit in a electronic transition and this can’t happen when j = 0 → j = 0
because there is no total angular momentum to re-orient to get a change of 1

David S. Latchman ©2009


Faraday’s Law 111
We can use this to calculate the individual resistances of the two wires in series.
2L
R1 = ρ (9.85.2)
A
L
R2 = ρ (9.85.3)
2A
Circuit 14 The Potential Difference across R1 is
R1
 
∆V = V1 (9.85.4)
R1 + R2
V1 is the potential difference across R1 . To get the Potential at A, we let

V1 = 8.0 − V (9.85.5)

Substituting eq. (9.85.5) into eq. (9.85.4), we get


R1

FT
 
8.0 − V = ∆V
R1 + R1
2
8.0 − V = (7)
3
1
V=3 (9.85.6)
3
RA
Answer: (B)

9.86 Faraday’s Law


Faraday’s Law tells us that the induced EMF is equal to the rate of change of Magnetic
Flux through a circuit. Thus

E=− (9.86.1)
dt
The Magnetic Flux is defined as
D

Z
Φ= B · dA (9.86.2)

At t = 0, the plane of the coil is in the Y direction. So

B = B0 sin ωt (9.86.3)

So Z
Φ= B0 sin ωt · πr2 (9.86.4)

Plugging this into Faraday’s Law, we have

E = ωπBr2 cos ωt (9.86.5)


14
Put Schematic of Resistors in Series here

©2009 David S. Latchman


112 GR0177 Exam Solutions
Each loop has an induced EMF as described in eq. (9.86.5). So n loops will have a total
EMF of nE. We can use Ohm’s Law to find the current,
nE
I=
R
nωBr2 π
= cos ωt
R
= 250π × 10−2 cos ωt (9.86.6)

Answer: (E)

9.87 Electric Potential

FT
The Electric Potential inside a charged sphere is zero as there is no Electric Field present.
So the Electric Force exerted on the positive charge, Q, by the sphere is also zero. So
we just have to consider the force exerted by the opposite sphere. The distance of the
charge, Q, from the center of the opposing sphere is

d
x = 10d − (9.87.1)
2
RA
The Electric Field is defined by Coulomb’s Law

1 qQ
F= (9.87.2)
4π0 x2

So the force exerted on the charge is

1 qQ
F=  
4π0 10d − d
2
qQ
=
D

(9.87.3)
361π0 d2

Both charges q and Q are the same so the force is repulsive, i.e. acts to the left.
Answer: (A)

9.88 Biot-Savart Law

The Biot-Savart Law is


µ0 I d` × r
dB = (9.88.1)
4π r2
Along the straight parts, the wire is parallel to the radius vector and hence the cross
product is zero; there is no magnetic contribution. But along the arc, the wire’s path is

David S. Latchman ©2009


Conservation of Angular Momentum 113
perpendicular to the radius vector, so there is contribution. Hence we will only look
at the arc. The length of arc, d`, is
d` = Rθ (9.88.2)

Substituting into the Biot-Savart Equation gives

θ
µ0 I Rθ
Z
B= dB = ·
0 4π R2
µ0 I θ
= (9.88.3)
4π R

Answer: (C)

9.89 Conservation of Angular Momentum

FT
We can find the Total Moment of Inertia of the child–merry-go-round system by using
the Parallel Axis Theorem.

Ii = Id + Ic
RA
1
= Md R2 + Mc R2
2 
1

= R2 Md + Mc
2
= 400 · 2.52 (9.89.1)

The final Moment of Inertia is just that of a disc. Moment of Inertia deals with how a
mass is distributed and taking the child as a point mass means that his rotation can be
ignored. Thus
D

1
I f = Md R2
2
= 100 · 2.52 (9.89.2)

Angular Momentum is conserved, thus

Ii ωi = I f ω f
Ii
⇒ ω f = ωi
If
140
= 2 = 2.8 rad/s (9.89.3)
100

Answer: (E)

©2009 David S. Latchman


114 GR0177 Exam Solutions
9.90 Springs in Series and Parallel
The Period of a Mass-Spring System is
r
ke
ω= (9.90.1)
m
Now it comes down to solving the effective spring constants for springs in series and
parallel. The addition of springs in series and parallel are the same as capacitors but if
you didn’t recall this you can solve the relationships.

9.90.1 Springs in Parallel

FT
In the parallel case, both springs extend by the same amount, x. The Forces on both
springs also add up sych that

F = F1 + F2
= −k1 x − k2 x (9.90.2)

This Parallel arrangement is the same as a Mass-Spring system with only one spring
of spring constant, ke . We have
F = −ke x
RA
(9.90.3)
(eq. (9.90.2)) = (eq. (9.90.3)), gives

ke = k1 + k2 (9.90.4)

9.90.2 Springs in Series

Springs in Series is a bit more challenging. For this case, we will assume light springs
such that the tension throughout the springs is constant. So we have
D

F = −k1 x1 = −k2 x2
k1 x2
⇒ = (9.90.5)
k2 x1
This is equivalent to a single Spring system where we again have a single Spring
System but
x = x1 + x2 (9.90.6)
Since the Forces are equal, we can say

kx = k2 x2
k (x1 + x2 ) = k2 x2
k1 k2
⇒k= (9.90.7)
k1 + k2

David S. Latchman ©2009


Cylinder rolling down an incline 115
We have thus shown that
1 1 1
= + (9.90.8)
k k1 k2
The period of the Mass-Spring Sytem for the Series Arrangement becomes

ke = k1 + k2 = 2k
r
m
Ts = 2π
ke
r
m
= 2π (9.90.9)
2k

The period for the Mass-Spring System in the Parallel arrangement becomes

FT
1 1 1
= +
k k k
2
=
k r
2m
Tp = 2π (9.90.10)
k

The ratio between Tp and Ts is


RA
q
2m
Tp 2π k
=
Ts
pm
2π 2k
=2 (9.90.11)

Answer: (E)15

9.91 Cylinder rolling down an incline


D

As the cylinder rolls down the hill, Gravitational Potential Energy is converted to
Translational Kinetic Energy and Rotational Kinectic Energy. This can be expressed as

PEgravity = KEtranslational + KErotational


1 1
mgh = mv2 + Iω2 (9.91.1)
2 2
Solving for I, we have
MR2  
I= 2gH − v2
(9.91.2)
v2
15
Springs are like Capacitors; when in parallel, their spring constants add and when in series, the
inverse of the total spring constant is the sum of the inverse of the individual ones.

©2009 David S. Latchman


116 GR0177 Exam Solutions
Plugging in for v, we get
!
7 8gH
I = MR 2
2gH −
8gH 7
3
= MR2 (9.91.3)
4
Answer: (B)

9.92 Hamiltonian of Mass-Spring System


The Hamiltonian of a System is
H =T+V (9.92.1)

FT
p2i
where Ti = 2m
and V = V(q). So the Hamiltonian is the sum of the kinetic energies of
the partiles and the energy stroed in the spring. Thus
( 2 2
1 p1 p2
)
H= + + k (` − `0 ) (9.92.2)
2 m m
Answer: (E)
RA
9.93 Radius of the Hydrogen Atom
The radial probability density for the ground state of the Hydrogen atom is found by
multiplying the square of the wavefunction by the spherical shell volume element.
Z
2
Pr = ψ0 dV (9.93.1)

The Volume of a sphere is V = 43 πr3 , so dV = 4πr2 . From the above equation we see
that
D

− 2r
dPr e a0
= · 4πr2 (9.93.2)
dr πa30
d2 Pr
We find the maxima and thus the most probable position by determining dr2
= 0.
Differentiating eq. (9.93.2) gives
d2 P 4 2 2
 
− a2r − a2r
= 2r · e 0 − r · · e 0 =0 (9.93.3)
dr2 a30 a0
Solving for r gives
r = a0 (9.93.4)
This is Bohr’s Radius which was found using semi-classical methods. In this case,
Schroedinger’s Equation confirms the first Bohr radius as the most probable radius
and more; the semi-classical Bohr’s Theory does not.
Answer: (C)

David S. Latchman ©2009


Perturbation Theory 117
9.94 Perturbation Theory

Perturbation Theory NOT FINISHED

9.95 Electric Field in a Dielectric

The Electric Field has magnitude

1 q σ
E= = (9.95.1)
4πκ0 r 2 κ0

FT
In a vacuum, κ = 1 and the strength of the Electric Field is E0 . So

E0
E= (9.95.2)
κ

Answer: (A)
RA
9.96 EM Radiation

Though the size of the sphere oscillates between R1 and R2 , the charge remains the
same. So the power radiated is zero.
Answer: (E)

9.97 Dispersion of a Light Beam


D

The Angular Spread of the light beam can be calculated by using Snell’s Law.

n1 sin θ1 = n2 sin θ2 (9.97.1)

For an Air-Glass system, this becomes

sin θ1 = n sin θ0 (9.97.2)

We know that for dispersion to take place then

θ0 = θ0 (λ) (9.97.3)
n = n(λ) (9.97.4)

©2009 David S. Latchman


118 GR0177 Exam Solutions
Differentiating eq. (9.97.2) with respect to λ, we have

d d
(sin θ) = (n sin θ0 )
dλ dλ
d
0= (n sin θ0 )

dn dθ0
= sin θ0 + n cos θ0
dλ dλ
dθ0 1 dn
⇒ = tan θ0
dλ n dλ
1 dn
∴ δθ =
0
tan θ δλ
0 0
(9.97.5)
n dλ

Answer: (E)

9.98 Average Energy of a Thermal System

FT
The thermodynamic total energy is simply the expected value of the energy; this is the
sum of the microstate energies weighed by their probabilities. This look like
RA
Ei e−Ei /kT
P
i
hEi = P −E /kT (9.98.1)
e i
i

Answer: (A)
D

9.99 Pair Production in vincinity of an electron

The familiar pair production reaction takes place in the Coulomb field of a massive
atom. As this nucleus is massive, we can ignore any recoil action of this spectator to
calculate the minimum energy needed for our photon. This time, our pair production
process takes place in the neighbourhood of an electron thus forcing us to take the
momenta and energies of all participants preset16
Our pair production process is

γ + e− −→ e− + e− + e+

16
This question was covered as an example question here.

David S. Latchman ©2009


Pair Production in vincinity of an electron 119
9.99.1 Solution 1

Momentum and Energy is conserved during the process. The energy of our photon is,
E . Conservation of Momentum shows us
E 3me v
= r (9.99.1)
c  2
v
1−
c
The left hand side of the equation is the momentum of our photon and the right hand
side is the momentum of all our electrons17 . We assume that their momenta is the same
for all. Energy conservation gives us
3me c2
E + me c2 = r  2 (9.99.2)
v

FT
1−
c
Dividing eq. (9.99.1) by eq. (9.99.2) gives us
E v
 
= (9.99.3)
E + me c2 c
Substituting eq. (9.99.3) into eq. (9.99.1) yields
E E E + me c2
RA
= 3me c (9.99.4)
c E + me c2 2E me c2 + (me c2 )2
p

After some very quick simplification, we get


E = 4me c2 (9.99.5)

9.99.2 Solution 2

You may find the above a bit calculation intensive; below is a somewhat quicker
D

solution but the principle is exactly the same. We use the same equations in a different
form. The total relativistic energy before our collision is
Ei = E + me c2 (9.99.6)
After collision, the relativistic energy of one electron is
2  2
E2e = pe c + me c2 (9.99.7)

We have three electrons so the final energy is


E f = 3Ee
q
2
= 3 pe c + (me c2 )2 (9.99.8)
17
I am using electrons to indicate both electrons & positrons. As they have the same rest mass we can
treat them the same. We don’t have to pay attention to their charges.

©2009 David S. Latchman


120 GR0177 Exam Solutions
Thus we have q
2
E + me c = 3
2
pe c + (me c2 )2 (9.99.9)
As momentum is conserved, we can say
p
pe = (9.99.10)
3
where p is the momentum of the photon, which happens to be

E = pc (9.99.11)

Substituting eq. (9.99.11) and eq. (9.99.10) into eq. (9.99.9) gives us

E = 4me c2 (9.99.12)

FT
Which is exactly what we got the first time we worked it out18 .
Answer: (D)

9.100 Michelson Interferometer


A fringe shift is registered when the movable mirror moves a full wavelength. So we
RA
can say
d = mλ (9.100.1)
where m is the number of fringes. If m g and mr are the number of fringes for green and
red light respectively, the wavelength of green light will be

mr λr
λg = (9.100.2)
mg

This becomes
D

(85865)(632.82)
λg =
100000
86000 · 630

100000
= 541. · · · (9.100.3)

Answer: (B)

18
The maximum wavelength of this works out to be

h λc
λ= = (9.99.13)
4me c 4
where λc is the Compton Wavelength

David S. Latchman ©2009


Appendix A
Constants & Important Equations

A.1 Constants

Constant
Speed of light in a vacuum
Gravitational Constant
FT
Symbol

G
c
Value
2.99 × 108 m/s
6.67 × 10−11 m3 /kg.s2
RA
Rest Mass of the electron me 9.11 × 10−31 kg
Avogadro’s Number NA 6.02 × 1023 mol-1
Universal Gas Constant R 8.31 J/mol.K
Boltzmann’s Constant k 1.38 × 10−23 J/K
Electron charge e 1.60 × 10−9 C
Permitivitty of Free Space 0 8.85 × 10−12 C2 /N.m2
Permeability of Free Space µ0 4π × 10−7 T.m/A
Athmospheric Pressure 1 atm 1.0 × 105 M/m2
Bohr Radius a0 0.529 × 10−10 m
D

Table A.1.1: Something

A.2 Vector Identities

A.2.1 Triple Products

A · (B × C) = B · (C × A) = C · (A × B) (A.2.1)
A × (B × C) = B (A · C) − C (A · B) (A.2.2)
122 Constants & Important Equations
A.2.2 Product Rules

∇ f g = f ∇g + g ∇ f
  
(A.2.3)
∇ (A · B) = A × (∇ × B) + B × (∇ × A) + (A · ∇) B + (B · ∇) A (A.2.4)
∇ · f A = f (∇ · A) + A · ∇ f
 
(A.2.5)
∇ · (A × B) = B · (∇ × A) − A · (∇ × B) (A.2.6)
∇ × f A = f (∇ × A) − A × ∇ f
 
(A.2.7)
∇ × (A × B) = (B · ∇) A − (A · ∇) B + A (∇ · B) − B (∇ · A) (A.2.8)

A.2.3 Second Derivatives

A.3 Commutators
∇ · (∇ × A) = 0
∇ × ∇f = 0


FT
∇ × (∇ × A) = ∇ (∇ · A) − ∇2 A
(A.2.9)
(A.2.10)
(A.2.11)
RA
A.3.1 Lie-algebra Relations

[A, A] = 0 (A.3.1)
[A, B] = −[B, A] (A.3.2)
[A, [B, C]] + [B, [C, A]] + [C, [A, B]] = 0 (A.3.3)
D

A.3.2 Canonical Commutator

[x, p] = i~ (A.3.4)

A.3.3 Kronecker Delta Function


(
0 if m , n;
δmn =
1 if m = n;
For a wave function Z
ψm (x)∗ ψn (x)dx = δmn (A.3.5)

David S. Latchman ©2009


Linear Algebra 123
A.4 Linear Algebra

A.4.1 Vectors

Vector Addition

The sum of two vectors is another vector

|αi + |βi = |γi (A.4.1)

Commutative
|αi + |βi = |βi + |αi (A.4.2)
Associative

FT
|αi + |βi + |γi = |αi + |βi + |γi
 
(A.4.3)
Zero Vector
|αi + |0i = |αi (A.4.4)
Inverse Vector
|αi + | − αi = |0i (A.4.5)
RA
D

©2009 David S. Latchman


124 Constants & Important Equations

FT
RA
D

David S. Latchman ©2009


Bibliography

[1] Wikipedia. Maxwell’s equations — wikipedia, the free encyclopedia, 2009. [Online;

FT
accessed 21-April-2009].
RA
D
Index

Angular Momentum, see Rotational Mo- Kepler’s Laws, see Celestial Mechanics
tion GR0177 Q3, 63
Kronecker Delta Function, 122
Bohr Model

FT
GR0177 Q18, 73 Linear Algebra, 123
GR0177 Q93, 116 Vectors, 123
Hydrogen Model, 43
Moment of Inertia, see Rotational Motion
Capacitors Newton’s Law of Gravitation, see Celestial
GR0177 Q10, 68 Mechanics
Celestial Mechanics, 10
Circular Orbits, 11 Oscillatory Motion, 4
RA
Escape Speed, 10 Coupled Harmonic Oscillators, 6
Kepler’s Laws, 11 Damped Motion, 5
Newton’s Law of Gravitation, 10 Kinetic Energy, 4
Orbits, 11 Potential Energy, 5
Potential Energy, 10 Simple Harmonic Motion Equation, 4
Circular Motion Small Oscillations, 5
GR0177 Q2, 62 Total Energy, 4
Circular Orbits, see Celestial Mechanics
Collisions Parallel Axis Theorem, see Rotational Mo-
GR0177 Q4, 64 tion
D

Commutators, 122 Pendulum


Canonical Commutators, 122 Simple
Kronecker Delta Function, 122 GR0177 Q1, 61
Lie-algebra Relations, 122 Rolling Kinetic Energy, see Rotational Mo-
Compton Effect, 46 tion
Counting Statistics, 59 Rotational Kinetic Energy, see Rotational
Motion
Doppler Effect, 8
Rotational Motion, 8
Equipartition Theorem Angular Momentum, 9
GR0177 Q5, 65 Moment of Inertia, 8
Parallel Axis Theorem, 9
Franck-Hertz Experiment, 49 Rolling Kinetic Energy, 9
Rotational Kinetic Energy, 8
Gravitation, see Celestial Mechanics Torque, 9
Index 127
Subject, 30
System of Particles, 10

Torque, see Rotational Motion

Vector Identities, 121


Product Rules, 122
Second Derivatives, 122
Triple Products, 121

FT
RA
D

©2009 David S. Latchman

S-ar putea să vă placă și